chapter 47 management of patients with Gastric and Duodenal disorders

Lakukan tugas rumah & ujian kamu dengan baik sekarang menggunakan Quizwiz!

A client is prescribed tetracycline to treat peptic ulcer disease. Which of the following instructions would the nurse give the client? a) "Be sure to wear sunscreen while taking this medicine." b) "Take the medication with milk." c) "Do not drive when taking this medication." d) "Expect a metallic taste when taking this medicine, which is normal."

"Be sure to wear sunscreen while taking this medicine." Explanation: Tetracycline may cause a photosensitivity reaction in clients. The nurse should caution the client to use sunscreen when taking this drug. Dairy products can reduce the effectiveness of tetracycline, so the nurse should not advise him or her to take the medication with milk. A metallic taste accompanies administration of metronidazole (Flagyl). Administration of tetracycline does not necessitate driving restrictions. Page 1264

A nurse is teaching a client with gastritis about the need to avoid the intake of caffeinated beverages. The client asks why this is so important. Which of the following explanations from the nurse would be most accurate? a) "Caffeine increases the fluid volume in your system, which irritates your digestive organs." b) "Caffeine intake can cause tears in your esophagus and intestines, which can lead to hemorrhage." c) "Caffeine can interfere with absorption of vitamin B12, which leads to anemia and further digestive problems." d) "Caffeine stimulates the central nervous system and thus gastric activity and secretions, which need to be minimized to promote recovery."

"Caffeine stimulates the central nervous system and thus gastric activity and secretions, which need to be minimized to promote recovery." Correct Explanation: Caffeine is a central nervous system stimulant that increases gastric activity and pepsin secretion. Caffeine is a diuretic that causes decreased fluid volume and potential dehydration. It does not lead to hemorrhage and does not interfere with absorption of vitamin B12. Page 1263

A patient with gastritis required hospital treatment for an exacerbation of symptoms and receives a subsequent diagnosis of pernicious anemia due to malabsorption. When planning the patient's continuing care in the home setting, what assessment question is most relevant? a) "Are you aware of your blood type?" b) "Are you going to be anywhere with strong sunlight in the next few months?" c) "Do any of your family members have training in first aid?" d) "Does anyone in your family have experience at giving injections?"

"Does anyone in your family have experience at giving injections?" Explanation: Patients with malabsorption of vitamin B12 need information about lifelong vitamin B12 injections; the nurse may instruct a family member or caregiver how to administer the injections or make arrangements for the patient to receive the injections from a health care provider. Questions addressing sun exposure, blood type and first aid are not directly relevant. Page 1278

The nurse determines that teaching for the client with peptic ulcer disease has been effective when the client states: a) "I have learned some relaxation strategies that decrease my stress." b) "I can buy whatever antacids are on sale because they all have the same effect." c) "I should continue my treatment regimen as long as I have pain." d) "I should stop all my medications if I develop any side effects."

"I have learned some relaxation strategies that decrease my stress." Correct Explanation: The nurse assists the client to identify stressful or exhausting situations. A hectic lifestyle and an irregular schedule may aggravate symptoms and interfere with regular meals taken in relaxed settings along with the regular administration of medications. The client may benefit from regular rest periods during the day, at least during the acute phase of the disease. Biofeedback, hypnosis, behavior modification, massage, or acupuncture may be helpful. Page 1270

The nurse determines that teaching for the client with peptic ulcer disease has been effective when the client states: a) "I can buy whatever antacids are on sale because they all have the same effect." b) "I should stop all my medications if I develop any side effects." c) "I have learned some relaxation strategies that decrease my stress." d) "I should continue my treatment regimen as long as I have pain."

"I have learned some relaxation strategies that decrease my stress." Explanation: The nurse assists the client to identify stressful or exhausting situations. A hectic lifestyle and an irregular schedule may aggravate symptoms and interfere with regular meals taken in relaxed settings along with the regular administration of medications. The client may benefit from regular rest periods during the day, at least during the acute phase of the disease. Biofeedback, hypnosis, behavior modification, massage, or acupuncture may be helpful. Page 1270

A patient who is obese has been unable to lose weight successfully using lifestyle modifications and has mentioned the possibility of using weight-loss medications. What should the nurse teach the patient about pharmacologic interventions for the treatment of obesity? a) "Medications can be helpful, but few people achieve and maintain their desired weight loss with medications alone." b) "There used to be a lot of hope that medications would help people lose weight, but it's been shown to be mostly a placebo effect." c) "Medications are rapidly become the preferred method of weight loss in people for whom diet and exercise have not worked." d) "Weight loss drugs have many side effects, and most doctors think they'll all be off the market in a few years."

"Medications can be helpful, but few people achieve and maintain their desired weight loss with medications alone." Explanation: Though antiobesity drugs help some patients lose weight, their use rarely results in loss of more than 10% of total body weight. Patients are consequently unlikely to attain their desired weight through medication alone. They are not predicted to disappear from the market and results are not attributed to a placebo effect. (less)

The nurse is conducting preoperative teaching to a patient prior to bariatric surgery. The nurse includes that the average weight loss of patients after bariatric surgery is which percent of their presurgical body weight? a) 5 to 15% b) 15 to 25% c) 25 to 35% d) 35 to 45%

25 to 35% Explanation: The nurse includes that the average weight loss of a patients after bariatric surgery is 25 to 35% of their presurgical body weight. - Page 1274

A client with peptic ulcer disease must begin triple medication therapy. For how long will the client follow this regimen? a) 7 to 9 days b) 10 to 14 days c) 4 to 6 days d) 15 to 20 days

10 to 14 days Correct Explanation: Recommended therapy for 10 to 14 days includes triple therapy with two antibiotics (eg, metronidazole [Flagyl] or amoxicillin [Amoxil] and clarithromycin [Biaxin]) plus a proton pump inhibitor (eg, lansoprazole [Prevacid], omeprazole [Prilosec], or rabeprazole [Aciphex]), or quadruple therapy with two antibiotics (metronidazole and tetracycline) plus a proton pump inhibitor and bismuth salts (Pepto-Bismol). Page 1267

Morbid obesity is defined as being how many pounds over the person's ideal body weight? a) 75 b) 90 c) 100 d) 50

100 Correct Explanation: Morbid obesity is a term applied to people who are more than two times their ideal body weight or whose body weight index (BMI) exceeds 30 kg/m2. Another definition of morbid obesity is body weight that is more than 100 pounds greater than the ideal body weight. Page 1272

Which of the following clients is at highest risk for peptic ulcer disease? a) A 31-year-old pregnant woman b) A 72-year-old grandfather of four c) A 19-year-old female college student d) A 52-year-old male accountant

A 52-year-old male accountant Explanation: Peptic ulcer disease occurs with the greatest frequency in people 40 and 60 years old. It is relatively uncommon in women of childbearing age, but it has been observed in children and even in infants. After menopause, the incidence of peptic ulcers in women is almost equal to that in men. Page 1265

A patient has come to the clinic complaining of pain just above her umbilicus. When assessing the patient, the nurse notes Sister Mary Joseph's nodules. The nurse should refer the patient to the primary care provider to be assessed for what health problem? a) Peptic ulcer disease b) Dumping syndrome c) Esophageal/gastric obstruction d) A GI malignancy

A GI malignancy Explanation: pg 1278 Palpable nodules around the umbilicus, called Sister Mary Joseph's nodules, are a sign of a GI malignancy, usually a gastric cancer. This would not be a sign of dumping syndrome, peptic ulcer disease, or esophageal/gastric obstruction. (

11. Which topic would be of importance for the nurse to include when teaching a patient about testing for possible metabolic syndrome? a. Blood glucose test b. Cardiac enzyme tests c. Postural blood pressures d. Resting electrocardiogram

ANS: A A fasting blood glucose test >100 mg/dL is one of the diagnostic criteria for metabolic syndrome. The other tests are not used to diagnose metabolic syndrome, although they may be used to check for cardiovascular complications of the disorder.

19. A patient is hospitalized with vomiting of "coffee-ground" emesis. The nurse will anticipate preparing the patient for a. endoscopy. b. angiography. c. gastric analysis testing. d. barium contrast studies.

ANS: A Endoscopy is the primary tool for visualization and diagnosis of upper gastrointestinal (GI) bleeding. Angiography is used only when endoscopy cannot be done because it is more invasive and has more possible complications. Gastric analysis testing may help with determining the cause of gastric irritation, but it is not used for acute GI bleeding. Barium studies are helpful in determining the presence of gastric lesions, but not whether the lesions are actively bleeding.

4. Which patient behavior indicates that an overweight patient has understood the nurse's teaching about the best exercise plan for weight loss? a. Walking for 40 minutes 6 or 7 days/week b. Lifting weights with friends 3 times/week c. Playing soccer for an hour on the weekend d. Running for 10 to 15 minutes 3 times/week

ANS: A Exercise should be done daily for 30 minutes to an hour. Exercising in highly aerobic activities for short bursts or only once a week is not helpful and may be dangerous in an individual who has not been exercising. Running may be appropriate, but a patient should start with an exercise that is less stressful and can be done for a longer period. Weight lifting is not as helpful as aerobic exercise in weight loss.

21. The health care provider orders intravenous (IV) ranitidine (Zantac) for a patient with gastrointestinal (GI) bleeding caused by peptic ulcer disease. When teaching the patient about the effect of the medication, which information will the nurse include? a. "Ranitidine decreases secretion of gastric acid." b. "Ranitidine neutralizes the acid in the stomach." c. "Ranitidine constricts the blood vessels in the stomach and decreases bleeding." d. "Ranitidine covers the ulcer with a protective material that promotes healing."

ANS: A Ranitidine is a histamine-2 (H2) receptor blocker, which decreases the secretion of gastric acid. The response beginning, "Ranitidine constricts the blood vessels" describes the effect of vasopressin. The response beginning "Ranitidine neutralizes the acid" describes the effect of antacids. And the response beginning "Ranitidine covers the ulcer" describes the action of sucralfate (Carafate).

30. Which information will be best for the nurse to include when teaching a patient with peptic ulcer disease (PUD) about dietary management of the disease? a."Avoid foods that cause pain after you eat them." b."High-protein foods are least likely to cause pain." c."You will need to remain on a bland diet indefinitely." d."You should avoid eating many raw fruits and vegetables."

ANS: A The best information is that each individual should choose foods that are not associated with postprandial discomfort. Raw fruits and vegetables may irritate the gastric mucosa, but chewing well seems to decrease this and some patients may tolerate these well. High-protein foods help to neutralize acid, but they also stimulate hydrochloric (HCl) acid secretion and may increase discomfort for some patients. Bland diets may be recommended during an acute exacerbation of PUD, but there is little scientific evidence to support their use.

27. A patient recovering from a gastrojejunostomy (Billroth II) for treatment of a duodenal ulcer develops dizziness, weakness, and palpitations about 20 minutes after eating. To avoid recurrence of these symptoms, the nurse teaches the patient to a. lie down for about 30 minutes after eating. b. choose foods that are high in carbohydrates. c. increase the amount of fluid intake with meals. d. drink sugared fluids or eat candy after each meal.

ANS: A The patient is experiencing symptoms of dumping syndrome, which may be reduced by lying down after eating. Increasing fluid intake and choosing high carbohydrate foods will increase the risk for dumping syndrome. Having a sweet drink or hard candy will correct the hypoglycemia that is associated with dumping syndrome but will not prevent dumping syndrome.

23. A patient with a bleeding duodenal ulcer has a nasogastric (NG) tube in place, and the health care provider orders 30 mL of aluminum hydroxide/magnesium hydroxide (Maalox) to be instilled through the tube every hour. To evaluate the effectiveness of this treatment, the nurse a.periodically aspirates and tests gastric pH. b.monitors arterial blood gas values on a daily basis. c.checks each stool for the presence of occult blood. d.measures the amount of residual stomach contents hourly.

ANS: A The purpose for antacids is to increase gastric pH. Checking gastric pH is the most direct way of evaluating the effectiveness of the medication. Arterial blood gases may change slightly, but this does not directly reflect the effect of antacids on gastric pH. Because the patient has upper gastrointestinal (GI) bleeding, occult blood in the stools will appear even after the acute bleeding has stopped. The amount of residual stomach contents is not a reflection of resolution of bleeding or of gastric pH.

2. After the nurse has completed teaching a patient about the recommended amounts of foods from animal and plant sources, which of these menu selections indicates that the initial instructions about diet have been understood? a. 3 oz of pork roast, a cup of corn, and a sliced tomato b.A chicken breast and a cup of tossed salad with nonfat dressing c.A 6 oz can of tuna mixed with nonfat mayonnaise and chopped celery d.3 oz of roast beef, 2 oz of low-fat cheese, and a half-cup of carrot sticks

ANS: A This selection is most consistent with the recommendation of the American Institute for Cancer Research that one third of the diet should be from animal sources and two thirds from plant source foods. The other choices all have higher ratios of animal origin foods to plant source foods than would be recommended.

15. A client has dumping syndrome after a partial gastrectomy. Which action by the nurse would be most helpful? a. Arrange a dietary consult. b. Increase fluid intake. c. Limit the client's foods. d. Make the client NPO.

ANS: A The client with dumping syndrome after a gastrectomy has multiple dietary needs. A referral to the registered dietitian will be extremely helpful. Food and fluid intake is complicated and needs planning. The client should not be NPO.

14. A client is in the bariatric clinic 1 month after having gastric bypass surgery. The client is crying and says "I didn't know it would be this hard to live like this." What response by the nurse is best? a. Assess the client's coping and support systems. b. Inform the client that things will get easier. c. Re-educate the client on needed dietary changes. d. Tell the client lifestyle changes are always hard.

ANS: A The nurse should assess this client's coping styles and support systems in order to provide holistic care. The other options do not address the client's distress.

1. A client is in the family practice clinic. Today the client weighs 186.4 pounds (84.7 kg). Six months ago the client weighed 211.8 pounds (96.2 kg). What action by the nurse is best? a. Ask the client if the weight loss was intentional. b. Determine if there are food allergies or intolerances. c. Perform a comprehensive nutritional assessment. d. Perform a rapid bedside blood glucose test.

ANS: A This client has had a 12% weight loss. The nurse first determines if the weight loss was intentional. If not, then the nurse proceeds to a comprehensive nutritional assessment. Food intolerances are part of this assessment. Depending on risk factors and other findings, a blood glucose test may be warranted.

15. A client has been prescribed lorcaserin (Belviq). What teaching is most appropriate? a. "Increase the fiber and water in your diet." b. "Reduce fat to less than 30% each day." c. "Report dry mouth and decreased sweating." d. "Lorcaserin may cause loose stools for a few days."

ANS: A This drug can cause constipation, so the client should increase fiber and water in the diet to prevent this from occurring. Reducing fat in the diet is important with orlistat. Lorcaserin can cause dry mouth but not decreased sweating. Loose stools are common with orlistat.

9. Which information will the nurse plan to include in discharge teaching for a patient after gastric bypass surgery? a.Avoid drinking fluids with meals. b.Choose high-fat foods for at least 30% of intake. c.Choose foods that are high in fiber to promote bowel function. d.Development of flabby skin can be prevented by daily exercise.

ANS: A pg1276 patient education Intake of fluids with meals tends to cause dumping syndrome and diarrhea. Food choices should be low in fat and fiber. Exercise does not prevent the development of flabby skin.

3. The student nurse learns about risk factors for gastric cancer. Which factors does this include? (Select all that apply.) a. Achlorhydria b. Chronic atrophic gastritis c. Helicobacter pylori infection d. Iron deficiency anemia e. Pernicious anemia

ANS: A, B, C, E Achlorhydria, chronic atrophic gastritis, H. pylori infection, and pernicious anemia are all risk factors for developing gastric cancer. Iron deficiency anemia is not a risk factor.

1. The student nurse studying stomach disorders learns that the risk factors for acute gastritis include which of the following? (Select all that apply.) a. Alcohol b. Caffeine c. Corticosteroids d. Fruit juice e. Nonsteroidal anti-inflammatory drugs (NSAIDs)

ANS: A, B, C, E Risk factors for acute gastritis include alcohol, caffeine, corticosteroids, and chronic NSAID use. Fruit juice is not a risk factor, although in some people it does cause distress. DIF: Remembering/Knowledge REF: 1127 KEY: Gastrointestinal disorders MSC: Integrated Process: Teaching/Learning NOT: Client Needs Category: Physiological Integrity: Physiological Adaptation

2. A client has a gastrointestinal hemorrhage and is prescribed two units of packed red blood cells. What actions should the nurse perform prior to hanging the blood? (Select all that apply.) a. Ask a second nurse to double-check the blood. b. Prime the IV tubing with normal saline. c. Prime the IV tubing with dextrose in water. d. Take and record a set of vital signs. e. Teach the client about reaction manifestations.

ANS: A, B, D, E Prior to starting a blood transfusion, the nurse asks another nurse to double-check the blood (and client identity), primes the IV tubing with normal saline, takes and records a baseline set of vital signs, and teaches the client about manifestations to report. The IV tubing is not primed with dextrose in water.

6. A client who had a partial gastrectomy has several expected nutritional problems. What actions by the nurse are best to promote better nutrition? (Select all that apply.) a. Administer vitamin B12 injections. b. Ask the provider about folic acid replacement. c. Educate the client on enteral feedings. d. Obtain consent for total parenteral nutrition. e. Provide iron supplements for the client.

ANS: A, B, E After gastrectomy, clients are at high risk for anemia due to vitamin B12 deficiency, folic acid deficiency, or iron deficiency. The nurse should provide supplements for all these nutrients. The client does not need enteral feeding or total parenteral nutrition.

. A nurse is preparing to administer pantoprazole (Protonix) intravenously. What actions by the nurse are most appropriate? (Select all that apply.) a. Administer the drug through a separate IV line. b. Infuse pantoprazole using an IV pump. c. Keep the drug in its original brown bag. d. Take vital signs frequently during infusion. e. Use an in-line IV filter when infusing.

ANS: A, B, E When infusing pantoprazole, use a separate IV line, a pump, and an in-line filter. A brown wrapper and frequent vital signs are not needed.

8. A nurse answers a client's call light and finds the client in the bathroom, vomiting large amounts of bright red blood. Which action should the nurse take first? a. Assist the client back to bed. b. Notify the provider immediately. c. Put on a pair of gloves. d. Take a set of vital signs.

ANS: C All of the actions are appropriate; however, the nurse should put on a pair of gloves first to avoid contamination with blood or body fluids.

1. The nurse understands that malnutrition can occur in hospitalized clients for several reasons. Which are possible reasons for this to occur? (Select all that apply.) a. Cultural food preferences b. Family bringing snacks c. Increased need for nutrition d. Need for NPO status e. Staff shortages

ANS: A, C, D, E Many factors increase the hospitalized client's risk for nutritional deficits. Cultural food preferences may make hospital food unpalatable. Ill clients have increased nutritional needs but may be NPO for testing or treatment, or have a loss of appetite from their illness. Staff shortages impact clients who need to be fed or assisted with meals. The family may bring snacks that are either healthy or unhealthy, so without further information, the nurse cannot assume the snacks are leading to malnutrition.

4. A client has dumping syndrome. What menu selections indicate the client understands the correct diet to manage this condition? (Select all that apply.) a. Canned unsweetened apricots b. Coffee cake c. Milk shake d. Potato soup e. Steamed broccoli

ANS: A, D Canned apricots and potato soup are appropriate selections as they are part of a high-protein, high-fat, low- to moderate-carbohydrate diet. Coffee cake and other sweets must be avoided. Milk products and sweet drinks such as shakes must be avoided. Gas-forming foods such as broccoli must also be avoided.

16. The nurse obtains these assessment data for a patient who has been taking orlistat (Xenical) for several months as part of a weight loss program. Which finding is most important to report to the health care provider? a. The patient frequently has liquid stools. b. The patient is pale and has many bruises. c. The patient is experiencing a plateau in weight loss. d. The patient complains of abdominal bloating after meals.

ANS: B Because orlistat blocks the absorption of fat-soluble vitamins, the patient may not be receiving an adequate amount of vitamin K, resulting in a decrease in clotting factors. Abdominal bloating and liquid stools are common side effects of orlistat and indicate that the nurse should remind the patient that fat in the diet may increase these side effects. Weight loss plateaus are normal during weight reduction.

3. When working with an obese patient who is enrolled in a behavior modification program, which nursing action is appropriate? a. Having the patient write down the caloric intake of each meal b. Asking the patient about situations that tend to increase appetite c. Encouraging the patient to eat small amounts throughout the day rather than having scheduled meals d. Suggesting that the patient have a reward, such as a piece of sugarless candy, after achieving a weight-loss goal

ANS: B Behavior modification programs focus on how and when the person eats and de-emphasize aspects such as calorie counting. Nonfood rewards are recommended for achievement of weight-loss goals. Patients are often taught to restrict eating to designated meals when using behavior modification.

10. Which assessment will the nurse do to help determine if an obese patient seen in the clinic has metabolic syndrome? a. Take the patient's apical pulse. b. Check the patient's blood pressure. c. Ask the patient about dietary intake. d. Dipstick the patient's urine for protein.

ANS: B Elevated blood pressure is one of the characteristics of metabolic syndrome. The other information also may be obtained by the nurse, but it will not assist with the diagnosis of metabolic syndrome.

22. The family member of a patient who has suffered massive abdominal trauma in an automobile accident asks the nurse why the patient is receiving famotidine (Pepcid). The nurse will explain that the medication will a. prevent aspiration of gastric contents. b. inhibit the development of stress ulcers. c. lower the chance for H. pylori infection. d. decrease the risk for nausea and vomiting.

ANS: B Famotidine is administered to prevent the development of physiologic stress ulcers, which are associated with a major physiologic insult such as massive trauma. Famotidine does not decrease nausea or vomiting, prevent aspiration, or prevent H. pylori infection.

17. A patient with peptic ulcer disease associated with the presence of Helicobacter pylori is treated with triple drug therapy. The nurse will plan to teach the patient about a.sucralfate (Carafate), nystatin (Mycostatin), and bismuth (Pepto-Bismol). b.amoxicillin (Amoxil), clarithromycin (Biaxin), and omeprazole (Prilosec). c.famotidine (Pepcid), magnesium hydroxide (Mylanta), and pantoprazole (Protonix). d.metoclopramide (Reglan), bethanechol (Urecholine), and promethazine (Phenergan).

ANS: B The drugs used in triple drug therapy include a proton pump inhibitor such as omeprazole and the antibiotics amoxicillin and clarithromycin. The other combinations listed are not included in the protocol for H. pyloriinfection.

6. A patient has been on a 1000-calorie diet with a daily exercise routine and a prescription for sibutramine (Meridia) for 10 weeks. Which information obtained by the nurse is important to report to the health care provider? a. The patient has not lost any weight for the last 2 weeks. b. The patient tells the nurse about occasional palpitations. c. The patient complains about having chronic constipation. d. The patient reports walking only 3 days during the last week.

ANS: B The patient may be experiencing an increase in heart rate caused by the sibutramine (Meridia) that should be evaluated further by the health care provider. Plateaus during weight loss programs are common. Chronic constipation may be a side effect of the sibutramine, and the nurse should instruct the patient in measures such as eating more high fiber foods and increasing fluid intake. The nurse should reinforce the need to exercise more frequently, but no additional intervention by the health care provider is necessary regarding the patient's activity level.

18. A nurse is caring for a morbidly obese client. What comfort measure is most important for the nurse to delegate to the unlicensed assistive personnel (UAP)? test bank a. Designating "quiet time" so the client can rest b. Ensuring siderails are not causing excess pressure c. Providing oral care before and after meals and snacks d. Relaying any reports of pain to the registered nurse

ANS: B All actions are good for client comfort, but when dealing with an obese client, the staff should take extra precautions, such as ensuring the siderails are not putting pressure on the client's tissues. The other options are appropriate for any client, and are not specific to obese clients.

6. An older female client has been prescribed esomeprazole (Nexium) for treatment of chronic gastric ulcers. What teaching is particularly important for this client? a. Check with the pharmacist before taking other medications. b. Increase intake of calcium and vitamin D. c. Report any worsening of symptoms to the provider. d. Take the medication as prescribed by the provider.

ANS: B All of this advice is appropriate for any client taking this medication. However, long-term use is associated with osteoporosis and osteoporosis-related fractures. This client is already at higher risk for this problem and should be instructed to increase calcium and vitamin D intake. The other options are appropriate for any client taking any medication and are not specific to the use of esomeprazole.

9. A client had an upper gastrointestinal hemorrhage and now has a nasogastric (NG) tube. What comfort measure may the nurse delegate to the unlicensed assistive personnel (UAP)? a. Lavaging the tube with ice water b. Performing frequent oral care c. Re-positioning the tube every 4 hours d. Taking and recording vital signs

ANS: B Clients with NG tubes need frequent oral care both for comfort and to prevent infection. Lavaging the tube is done by the nurse. Re-positioning the tube, if needed, is also done by the nurse. The UAP can take vital signs, but this is not a comfort measure.

16. Several nurses have just helped a morbidly obese client get out of bed. One nurse accesses the client's record because "I just have to know how much she weighs!" What action by the client's nurse is most appropriate? a. Make an anonymous report to the charge nurse. b. State "That is a violation of client confidentiality." c. Tell the nurse "Don't look; I'll tell you her weight." d. Walk away and ignore the other nurse's behavior.

ANS: B Ethical practice requires the nurse to speak up and tell the other nurse that he or she is violating client confidentiality rules. The other responses do not address this concern.

13. A morbidly obese client is admitted to a community hospital that does not typically care for bariatric-sized clients. What action by the nurse is most appropriate? a. Assess the client's readiness to make lifestyle changes. b. Ensure adequate staff when moving the client. c. Leave siderails down to prevent pressure ulcers. d. Reinforce the need to be sensitive to the client.

ANS: B Many hospitals that see bariatric-sized clients have appropriate equipment for this population. A hospital that does not typically see these clients is less likely to have appropriate equipment, putting staff and client safety at risk. The nurse ensures enough staffing is available to help with all aspects of mobility. It may or may not be appropriate to assess the client's willingness to make lifestyle changes. Leaving the siderails down may present a safety hazard. The staff should be sensitive to this client's situation, but safety takes priority.

19. A client is awaiting bariatric surgery in the morning. What action by the nurse is most important? a. Answering questions the client has about surgery b. Beginning venous thromboembolism prophylaxis c. Informing the client that he or she will be out of bed tomorrow d. Teaching the client about needed dietary changes

ANS: B Morbidly obese clients are at high risk of venous thromboembolism and should be started on a regimen to prevent this from occurring as a priority. Answering questions about the surgery is done by the surgeon. Teaching is important, but safety comes first.

2. A client has a pyloric obstruction and reports sudden muscle weakness. What action by the nurse takes priority? a. Document the findings in the chart. b. Request an electrocardiogram (ECG). c. Facilitate a serum potassium test. d. Place the client on bedrest.

ANS: B Pyloric stenosis can lead to hypokalemia, which is manifested by muscle weakness. The nurse first obtains an ECG because potassium imbalances can lead to cardiac dysrhythmias. A potassium level is also warranted, as is placing the client on bedrest for safety. Documentation should be thorough, but none of these actions takes priority over the ECG.

11. A client has a recurrence of gastric cancer and is in the gastrointestinal clinic crying. What response by the nurse is most appropriate? a. "Do you have family or friends for support?" b. "I'd like to know what you are feeling now." c. "Well, we knew this would probably happen." d. "Would you like me to refer you to hospice?"

ANS: B The nurse assesses the client's emotional state with open-ended questions and statements and shows a willingness to listen to the client's concerns. Asking about support people is very limited in nature, and "yes-or-no" questions are not therapeutic. Stating that this was expected dismisses the client's concerns. The client may or may not be ready to hear about hospice, and this is another limited, yes-or-no question.

16. An older client has gastric cancer and is scheduled to have a partial gastrectomy. The family does not want the client told about her diagnosis. What action by the nurse is best? a. Ask the family why they feel this way. b. Assess family concerns and fears. c. Refuse to go along with the family's wishes. d. Tell the family that such secrets cannot be kept.

ANS: B The nurse should use open-ended questions and statements to fully assess the family's concerns and fears. Asking "why" questions often puts people on the defensive and is considered a barrier to therapeutic communication. Refusing to follow the family's wishes or keep their confidence will not help move this family from their position and will set up an adversarial relationship.

12. A client with peptic ulcer disease asks the nurse about taking slippery elm supplements. What response by the nurse is best? a. "Slippery elm has no benefit for this problem." b. "Slippery elm is often used for this disorder." c. "There is no evidence that this will work." d. "You should not take any herbal remedies."

ANS: B There are several complementary and alternative medicine regimens that are used for gastritis and peptic ulcer disease. Most have been tested on animals but not humans. Slippery elm is a common supplement used for this disorder.

1. The nurse is caring for a client with peptic ulcer disease who reports sudden onset of sharp abdominal pain. On palpation, the client's abdomen is tense and rigid. What action takes priority? a. Administer the prescribed pain medication. b. Notify the health care provider immediately. c. Percuss all four abdominal quadrants. d. Take and document a set of vital signs.

ANS: B This client has manifestations of a perforated ulcer, which is an emergency. The priority is to get the client medical attention. The nurse can take a set of vital signs while someone else calls the provider. The nurse should not percuss the abdomen or give pain medication since the client may need to sign consent for surgery.

4. A client's small-bore feeding tube has become occluded after the nurse administered medications. What actions by the nurse are best? (Select all that apply.) a. Attempt to dissolve the clog by instilling a cola product. b. Determine if any of the medications come in liquid form. c. Flush the tube before and after administering medications. d. Mix all medications in the formula and use a feeding pump. e. Try to flush the tube with 30 mL of water and gentle pressure.

ANS: B, C, E If the tube is obstructed, use a 50-mL syringe and gentle pressure to attempt to open the tube. Cola products should not be used unless water is not effective. To prevent future problems, determine if any of the medications can be dispensed in liquid form and flush the tube with water before and after medication administration. Do not mix medications with the formula.

3. A nurse is designing a community education program to meet the Healthy People 2020 objectives for nutrition and weight status. What information about these goals does the nurse use to plan this event? (Select all that apply.) a. Decrease the amount of fruit to 1.1 cups/1000 calories. b. Increase the amount of vegetables to 1.1 cups/1000 calories. c. Increase the number of adults at a healthy weight by 25%. d. Reduce the number of adults who are obese by 10%. e. Reduce the consumption of saturated fat by nearly 10%.

ANS: B, D, E Some of the goals in this initiative include increasing fruit consumption to 0.9 cups/1000 calories, increasing vegetable intake to 1.1 cups/1000 calories, increasing the number of people at a healthy weight by 10%, decreasing the number of adults who are obese by 10%, and reducing the consumption of saturated fats by 9.5%.

26. The nurse implements discharge teaching for a patient following a gastroduodenostomy for treatment of a peptic ulcer. Which patient statement indicates that the teaching has been effective? a. "Persistent heartburn is expected after surgery." b. "I will try to drink liquids along with my meals." c. "Vitamin supplements may be needed to prevent problems with anemia." d. "I will need to choose foods that are low in fat and high in carbohydrate."

ANS: C Cobalamin deficiency may occur after partial gastrectomy, and the patient may need to receive cobalamin via injections or nasal spray. Foods that have moderate fat and low carbohydrate should be chosen to prevent dumping syndrome. Ingestion of liquids with meals is avoided to prevent dumping syndrome. Although peptic ulcer disease may recur, persistent heartburn is not expected after surgery and the patient should call the health care provider if this occurs.

19. In planning preoperative teaching for a patient undergoing a Roux-en-Y gastric bypass as treatment for morbid obesity, the nurse places the highest priority on a.demonstrating passive range-of-motion exercises to the legs. b.discussing the necessary postoperative modifications in lifestyle. c.teaching the patient proper coughing and deep breathing techniques. d.educating the patient about the postoperative presence of a nasogastric (NG) tube.

ANS: C Coughing and deep breathing can prevent major postoperative complications such as carbon monoxide retention and hypoxemia. Information about passive range of motion, the NG tube, and postoperative modifications in lifestyle also will be discussed, but avoidance of respiratory complications is the priority goal after surgery.

14. Which nursing action included in the plan of care for a patient who is being admitted for bariatric surgery can the nurse delegate to nursing assistive personnel (NAP)? a.Demonstrate use of the incentive spirometer. b.Plan methods for bathing and turning the patient. c.Assist with IV insertion by holding adipose tissue out of the way. d.Develop strategies to provide privacy and decrease embarrassment.

ANS: C NAP can assist with IV placement by assisting with patient positioning or holding skinfolds aside. Planning for care and patient teaching require RN level education and scope of practice.

5. When the health care provider in the outpatient clinic is considering prescribing sibutramine (Meridia) for a patient, which patient information is most important for the nurse to discuss with the provider? a. The patient has a permanent pacemaker. b. The patient's goal is to lose 90 lb (41 kg). c. The patient's blood pressure is usually 135-145/85-95. d. The patient used fenfluramine (Pondimin) in the past for weight loss.

ANS: C Side effects of sibutramine (Meridia) include hypertension. A permanent pacemaker and a history of fenfluramine use are not contraindications for sibutramine. Sibutramine is prescribed for patients who have large weight loss goals.

29. The health care provider prescribes antacids and sucralfate (Carafate) for treatment of a patient's peptic ulcer. The nurse will teach the patient to take a. antacids 30 minutes before the sucralfate. b. sucralfate at bedtime and antacids before meals. c. antacids after eating and sucralfate 30 minutes before eating. d. sucralfate and antacids together 30 minutes before each meal.

ANS: C Sucralfate is most effective when the pH is low and should not be given with or soon after antacid. Antacids are most effective when taken after eating. Administration of sucralfate 30 minutes before eating and antacids just after eating will ensure that both drugs can be most effective. The other regimens will decrease the effectiveness of the medications.

15. A patient who has been successfully losing 1 lb weekly for several months is weighed at the clinic and has not lost any weight for the last month. The nurse should first a. review the diet and exercise guidelines with the patient. b. instruct the patient to weigh weekly and record the weights. c. ask the patient whether there have been any changes in exercise or diet patterns. d. discuss the possibility that the patient has reached a temporary weight loss plateau.

ANS: C The initial nursing action should be assessment of any reason for the change in weight loss. The other actions may be needed, but further assessment is required before any interventions are planned or implemented.

12. What specific information will the nurse include in patient teaching for an overweight patient who is starting a weight loss plan? a. Weigh yourself at the same time every morning. b. Start dieting with a 600- to 800-calorie diet for rapid weight loss. c. Low carbohydrate diets lead to rapid weight loss but are difficult to maintain. d. Weighing all foods on a scale is necessary to choose appropriate portion sizes.

ANS: C The restrictive nature of fad diets makes the weight loss achieved by the patient more difficult to maintain. Portion size can be estimated in other ways besides weighing. Severely calorie-restricted diets are not necessary for patients in the overweight category of obesity and need to be closely supervised. Patients should weigh weekly rather than daily.

4. A client with a bleeding gastric ulcer is having a nuclear medicine scan. What action by the nurse is most appropriate? a. Assess the client for iodine or shellfish allergies. b. Educate the client on the side effects of sedation. c. Inform the client a second scan may be needed. d. Teach the client about bowel preparation for the scan.

ANS: C A second scan may be performed in 1 to 2 days to see if interventions have worked. The nuclear medicine scan does not use iodine-containing contrast dye or sedation. There is no required bowel preparation.

12. A client just returned to the surgical unit after a gastric bypass. What action by the nurse is the priority? a. Assess the client's pain. b. Check the surgical incision. c. Ensure an adequate airway. d. Program the morphine pump.

ANS: C All actions are appropriate care measures for this client; however, airway is always the priority. Bariatric clients tend to have short, thick necks that complicate airway management.

13. A nurse is teaching a client about magnesium hydroxide with aluminum hydroxide (Maalox). What instruction is most appropriate? test quesiton a. "Aspirin must be avoided." b. "Do not worry about black stools." c. "Report diarrhea to your provider." d. "Take 1 hour before meals."

ANS: C Maalox can cause hypermagnesemia, which causes diarrhea, so the client should be taught to report this to the provider. Aspirin is avoided with bismuth sulfate (Pepto-Bismol). Black stools can be caused by Pepto-Bismol. Maalox should be taken after meals.

14. For which client would the nurse suggest the provider not prescribe misoprostol (Cytotec)? a. Client taking antacids b. Client taking antibiotics c. Client who is pregnant d. Client over 65 years of age

ANS: C Misoprostol can cause abortion, so pregnant women should not take this drug. The other clients have no contraindications to taking misoprostol.

7. The nurse caring for clients with gastrointestinal disorders should understand that which category best describes the mechanism of action of sucralfate (Carafate)? a. Gastric acid inhibitor b. Histamine receptor blocker c. Mucosal barrier fortifier d. Proton pump inhibitor

ANS: C Sucralfate is a mucosal barrier fortifier (protector). It is not a gastric acid inhibitor, a histamine receptor blocker, or a proton pump inhibitor.

17. A nurse attempted to assist a morbidly obese client back to bed and had immediate pain in the lower back. What action by the nurse is most appropriate? a. Ask another nurse to help next time. b. Demand better equipment to use. c. Fill out and file a variance report. d. Refuse to assist the client again.

ANS: C The nurse should complete a variance report per agency policy. Asking another nurse to help and requesting better equipment are both good ideas, but the nurse may have an injury that needs care. It would be unethical to refuse to care for this client again.

3. A client with peptic ulcer disease is in the emergency department and reports the pain has gotten much worse over the last several days. The client's blood pressure when lying down was 122/80 mm Hg and when standing was 98/52 mm Hg. What action by the nurse is most appropriate? a. Administer ibuprofen (Motrin). b. Call the Rapid Response Team. c. Start a large-bore IV with normal saline. d. Tell the client to remain lying down.

ANS: C This client has orthostatic changes to the blood pressure, indicating fluid volume loss. The nurse should start a large-bore IV with isotonic solution. Ibuprofen will exacerbate the ulcer. The Rapid Response Team is not needed at this point. The client should be put on safety precautions, which includes staying in bed, but this is not the priority.

5. A nurse working with a client who has possible gastritis assesses the client's gastrointestinal system. Which findings indicate a chronic condition as opposed to acute gastritis? (Select all that apply.) a. Anorexia b. Dyspepsia c. Intolerance of fatty foods d. Pernicious anemia e. Nausea and vomiting

ANS: C, D Intolerance of fatty or spicy foods and pernicious anemia are signs of chronic gastritis. Anorexia and nausea/vomiting can be seen in both conditions. Dyspepsia is seen in acute gastritis.

2. A nurse has delegated feeding a client to an unlicensed assistive personnel (UAP). What actions does the nurse include in the directions to the UAP? (Select all that apply.) a. Allow 30 minutes for eating so food doesn't get spoiled. b. Assess the client's mouth while providing premeal oral care. c. Ensure warm and cold items stay at appropriate temperatures. d. Remove bedpans, soiled linens, and other unpleasant items. e. Sit with the client, making the atmosphere more relaxed.

ANS: C, D, E The UAP should make sure food items remain at the appropriate temperatures for maximum palatability. Removing items such as bedpans, urinals, or soiled linens helps make the atmosphere more conducive to eating. The UAP should sit, not stand, next to the client to promote a relaxing experience. The client, especially older clients who tend to eat more slowly, should not be rushed. Assessment is done by the nurse.

28. A patient who requires daily use of a nonsteroidal anti-inflammatory drug (NSAID) for management of severe rheumatoid arthritis has recently developed melena. The nurse will anticipate teaching the patient about test bank a.substitution of acetaminophen (Tylenol) for the NSAID. b.use of enteric-coated NSAIDs to reduce gastric irritation. c.reasons for using corticosteroids to treat the rheumatoid arthritis. d.the benefits of misoprostol (Cytotec) in protecting the gastrointestinal (GI) mucosa.

ANS: D Misoprostol, a prostaglandin analog, reduces acid secretion and incidence of upper GI bleeding associated with NSAID use. Enteric coating of NSAIDs does not reduce the risk for GI bleeding. Corticosteroids increase the risk for ulcer development and will not be substituted for NSAIDs for this patient. Acetaminophen will not be effective in treating the patient's rheumatoid arthritis.

1. The nurse is developing a weight loss plan for a 21-year-old patient who is morbidly obese. Which statement by the nurse is most likely to help the patient in losing weight on the planned 1000-calorie diet? a."It will be necessary to change lifestyle habits permanently to maintain weight loss." b."You will decrease your risk for future health problems such as diabetes by losing weight now." c."Most of the weight that you lose during the first weeks of dieting is water weight rather than fat." d."You are likely to start to notice changes in how you feel with just a few weeks of diet and exercise."

ANS: D Motivation is a key factor in successful weight loss and a short-term outcome provides a higher motivation. A 21-year-old patient is unlikely to be motivated by future health problems. Telling a patient that the initial weight loss is water will be discouraging, although this may be correct. Changing lifestyle habits is necessary, but this process occurs over time and discussing this is not likely to motivate the patient.

35. Which of these nursing actions should the RN working in the emergency department delegate to nursing assistive personnel who help with the care of a patient who has been admitted with nausea and vomiting? a. Auscultate the bowel sounds. b. Assess for signs of dehydration. c. Ask the patient what precipitated the nausea. d. Assist the patient with oral care after vomiting.

ANS: D Oral care is included in nursing assistive personnel education and scope of practice. The other actions are all assessments that require more education and a higher scope of nursing practice.

7. A few months after bariatric surgery, a 62-year-old patient tells the nurse, "My skin is hanging in folds. I think I need cosmetic surgery." Which response by the nurse is most appropriate? a. "Perhaps you would like to talk to a counselor about your body image." b. "The important thing is that your weight loss is improving your health." c. "The skinfolds will gradually disappear once most of the weight is lost." d. "Cosmetic surgery is certainly a possibility once your weight has stabilized."

ANS: D Reconstructive surgery may be used to eliminate excess skinfolds after at least a year has passed since the surgery. Skinfolds may not disappear over time, especially in older patients. The response, "The important thing is that your weight loss is improving your health" ignores the patient's concerns about appearance and implies that the nurse knows what is important. Whereas it may be helpful for the patient to talk to a counselor, it is more likely to be helpful to know that cosmetic surgery is available.

8. A patient returns to the surgical nursing unit following a vertical banded gastroplasty with a nasogastric tube to low, intermittent suction and a patient-controlled analgesia (PCA) machine for pain control. Which nursing action should be included in the postoperative plan of care? a.Irrigate the nasogastric (NG) tube frequently with normal saline. b Offer sips of sweetened liquids at frequent intervals. c.Remind the patient that PCA use may slow the return of bowel function. d.Support the surgical incision during patient coughing and turning in bed.

ANS: D The incision should be protected from strain to decrease the risk for wound dehiscence. The patient should be encouraged to use the PCA since pain control will improve cough effort and patient mobility. NG irrigation may damage the suture line or overfill the stomach pouch. Sugar-free clear liquids are offered during the immediate postoperative time to decrease the risk for dumping syndrome.

17. When developing a weight reduction plan for an obese patient who wants to lose weight, which question should the nurse ask first? a. "Which food types do you like best?" b. "How long have you been overweight?" c. "What kind of physical activities do you enjoy?" d. "What factors do you think led to your obesity?"

ANS: D The nurse should obtain information about the patient's perceptions of the reasons for the obesity to develop a plan individualized to the patient. The other information also will be obtained from the patient, but the patient is more likely to make changes when the patient's beliefs are considered in planning.

13. Which of these patients in the clinic will the nurse plan to teach about risks associated with obesity? a. Patient who has a BMI of 18 kg/m2 b. Patient with a waist circumference 34 inches (86 cm) c. Patient who has a body mass index (BMI) of 24 kg/m2 d. Patient whose waist measures 30 in (75 cm) and hips measure 34 in (85 cm)

ANS: D The waist-to-hip ratio for this patient is 0.88, which exceeds the recommended level of <0.80. A BMI of 24 kg/m2 is normal. Health risks associated with obesity increase in women with a waist circumference larger than 35 in (89 cm) and men with a waist circumference larger than 40 in (102 cm). A patient with a BMI of 18 kg/m2 is considered underweight.

18. On the first postoperative day the nurse is caring for a patient who has had a Roux-en-Y gastric bypass procedure. Which assessment finding should be reported immediately to the surgeon? a.Use of patient-controlled analgesia (PCA) several times an hour for pain b.Irritation and skin breakdown in skinfolds c.Bilateral crackles audible at both lung bases d.Emesis of bile-colored fluid past the nasogastric (NG) tube

ANS: D Vomiting with an NG tube in place indicates that the NG tube needs to be repositioned by the surgeon to avoid putting stress on the gastric sutures. The nurse should implement actions to decrease skin irritation and have the patient cough and deep breathe, but these do not indicate a need for rapid notification of the surgeon. Frequent PCA use after bariatric surgery is expected.

10. A client is scheduled for a total gastrectomy for gastric cancer. What preoperative laboratory result should the nurse report to the surgeon immediately? a. Albumin: 2.1 g/dL b. Hematocrit: 28% c. Hemoglobin: 8.1 mg/dL d. International normalized ratio (INR): 4.2

ANS: D An INR as high as 4.2 poses a serious risk of bleeding during the operation and should be reported. The albumin is low and is an expected finding. The hematocrit and hemoglobin are also low, but this is expected in gastric cancer.

11. A client asks the nurse about drugs for weight loss. What response by the nurse is best? a. "All weight-loss drugs can cause suicidal ideation." b. "No drugs are currently available for weight loss." c. "Only over-the-counter medications are available." d. "There are three drugs currently approved for this."

ANS: D There are three drugs available by prescription for weight loss, including orlistat (Xenical), lorcaserin (Belviq), and phentermine-topiramate (Qsymia). Suicidal thoughts are possible with lorcaserin and phentermine-topiramate. Orlistat is also available in a reduced-dose over-the-counter formulation.

5. A client is being taught about drug therapy for Helicobacter pylori infection. What assessment by the nurse is most important? a. Alcohol intake of 1 to 2 drinks per week b. Family history of H. pylori infection c. Former smoker still using nicotine patches d. Willingness to adhere to drug therapy

ANS: D Treatment for this infection involves either triple or quadruple drug therapy, which may make it difficult for clients to remain adherent. The nurse should assess the client's willingness and ability to follow the regimen. The other assessment findings are not as critical.

A client reports to the clinic, stating that she rapidly developed headache, abdominal pain, nausea, hiccuping, and fatigue about 2 hours ago. For dinner, she ate buffalo chicken wings and beer. Which of the following medical conditions is most consistent with the client's presenting problems? a) Duodenal ulcer b) Acute gastritis c) Gastric cancer d) Gastric ulcer

Acute gastritis Explanation: The client with acute gastritis may have a rapid onset of symptoms, including abdominal discomfort, headache, lassitude, nausea, anorexia, vomiting, and hiccuping, which can last from a few hours to a few days. Acute gastritis is often caused by dietary indiscretion--a person eats food that is irritating, too highly seasoned, or contaminated with disease-causing microorganisms. Page 1262

The nurse is assessing an 80-year-old patient for signs and symptoms of gastric cancer. The nurse differentiates which of the following to be a sign/symptom of gastric cancer in the geriatric patient, but not in a patient under the age of 75? a) Agitation b) Hepatomegalia c) Abdominal mass d) Ascites

Agitation Explanation: The nurse differentiates that agitation, along with confusion and restlessness, may be the only signs/symptoms seen of gastric cancer in the older patient. Abdominal mass, hepatomegaly, and ascites may all be signs/symptoms of advanced gastric cancer. Page 1279

40. A patient has come to the clinic complaining of pain just above her umbilicus. When assessing the patient, the nurse notes Sister Mary Joseph's nodules. The nurse should refer the patient to the primary care provider to be assessed for what health problem? A) A GI malignancy B) Dumping syndrome C) Peptic ulcer disease D) Esophageal/gastric obstruction

Ans: A Feedback: Palpable nodules around the umbilicus, called Sister Mary Joseph's nodules, are a sign of a GI malignancy, usually a gastric cancer. This would not be a sign of dumping syndrome, peptic ulcer disease, or esophageal/gastric obstruction.

38. A patient with gastritis required hospital treatment for an exacerbation of symptoms and receives a subsequent diagnosis of pernicious anemia due to malabsorption. When planning the patient's continuing care in the home setting, what assessment question is most relevant? A) Does anyone in your family have experience at giving injections? B) Are you going to be anywhere with strong sunlight in the next few months? C) Are you aware of your blood type? D) Do any of your family members have training in first aid?

Ans: A Feedback: Patients with malabsorption of vitamin B12 need information about lifelong vitamin B12injections; the nurse may instruct a family member or caregiver how to administer the injections or make arrangements for the patient to receive the injections from a health care provider. Questions addressing sun exposure, blood type and first aid are not directly relevant.

8. A nurse in the postanesthesia care unit admits a patient following resection of a gastric tumor. Following immediate recovery, the patient should be placed in which position to facilitate patient comfort and gastric emptying? A) Fowler's B) Supine C) Left lateral D) Left Sim's

Ans: A Feedback: Positioning the patient in a Fowler's position postoperatively promotes comfort and facilitates emptying of the stomach following gastric surgery. Any position that involves lying down delays stomach emptying and is not recommended for this type of patient. Supine positioning and the left lateral (left Sim's) position do not achieve this goal.

29. A nurse is performing the admission assessment of a patient whose high body mass index (BMI) corresponds to class III obesity. In order to ensure empathic and patient-centered care, the nurse should do which of the following? A) Examine one's own attitudes towards obesity in general and the patient in particular. B) Dialogue with the patient about the lifestyle and psychosocial factors that resulted in obesity. C) Describe one's own struggles with weight gain and weight loss to the patient. D) Elicit the patient's short-term and long-term goals for weight loss.

Ans: A Feedback: Studies suggest that health care providers, including nurses, harbor negative attitudes towards obese patients. Nurses have a responsibility to examine these attitudes and change them accordingly. This is foundational to all other areas of assessing this patient.

35. A patient has received a diagnosis of gastric cancer and is awaiting a surgical date. During the preoperative period, the patient should adopt what dietary guidelines? A) Eat small, frequent meals with high calorie and vitamin content. B) Eat frequent meals with an equal balance of fat, carbohydrates, and protein. C) Eat frequent, low-fat meals with high protein content. D) Try to maintain the pre-diagnosis pattern of eating.

Ans: A Feedback: The nurse encourages the patient to eat small, frequent portions of nonirritating foods to decrease gastric irritation. Food supplements should be high in calories, as well as vitamins A and C and iron, to enhance tissue repair.

34. A patient has recently received a diagnosis of gastric cancer; the nurse is aware of the importance of assessing the patient's level of anxiety. Which of the following actions is most likely to accomplish this? A) The nurse gauges the patient's response to hypothetical outcomes. B) The patient is encouraged to express fears openly. C) The nurse provides detailed and accurate information about the disease. D) The nurse closely observes the patient's body language.

Ans: B Feedback: Encouraging the patient to discuss his or her fears and anxieties is usually the best way to assess a patient's anxiety. Presenting hypothetical situations is a surreptitious and possibly inaccurate way of assessing anxiety. Observing body language is part of assessment, but it is not the complete assessment. Presenting information may alleviate anxiety for some patients, but it is not an assessment.

30. A patient has been prescribed orlistat (Xenical) for the treatment of obesity. When providing relevant health education for this patient, the nurse should ensure the patient is aware of what potential adverse effect of treatment? A) Bowel incontinence B) Flatus with oily discharge C) Abdominal pain D) Heat intolerance

Ans: B Feedback: Side effects of orlistat include increased frequency of bowel movements, gas with oily discharge, decreased food absorption, decreased bile flow, and decreased absorption of some vitamins. This drug does not cause bowel incontinence, abdominal pain, or heat intolerance.

21. A nurse is providing anticipator guidance to a patient who is preparing for bariatric surgery. The nurse learns that the patient is anxious about numerous aspects of the surgery. What intervention is most appropriate to alleviate the patient's anxiety? A) Emphasize the fact that bariatric surgery has a low risk of complications. B) Encourage the patient to focus on the benefits of the surgery. C) Facilitate the patient's contact with a support group. D) Obtain an order for a PRN benzodiazepine.

Ans: C Feedback: Support groups can be highly beneficial in relieving preoperative and postoperative anxiety and in promoting healthy coping. This is preferable to antianxiety medications. Downplaying the risks of surgery or focusing solely on the benefits is a simplistic and patronizing approach.

The nurse is assessing a patient with progressive gastric cancer. The nurse anticipates that the assessment will reveal which of the following findings? a) Bloating after meals b) Increased appetite c) Weight gain d) Abdominal pain below umbilicus

Bloating after meals Correct Explanation: Symptoms of progressive disease include bloating after meals, weight loss, abdominal pain above the umbilicus, and loss or decrease in appetite. Page 1278

37. A nurse is caring for a patient hospitalized with an exacerbation of chronic gastritis. What health promotion topic should the nurse emphasize? A) Strategies for maintaining an alkaline gastric environment B) Safe technique for self-suctioning C) Techniques for positioning correctly to promote gastric healing D) Strategies for avoiding irritating foods and beverages

Ans: D Feedback: Measures to help relieve pain include instructing the patient to avoid foods and beverages that may be irritating to the gastric mucosa and instructing the patient about the correct use of medications to relieve chronic gastritis. An alkaline gastric environment is neither possible nor desirable. There is no plausible need for self-suctioning. Positioning does not have a significant effect on the presence or absence of gastric healing.

9. A community health nurse is preparing for an initial home visit to a patient discharged following a total gastrectomy for treatment of gastric cancer. What would the nurse anticipate that the plan of care is most likely to include? A) Enteral feeding via gastrostomy tube (G tube) B) Gastrointestinal decompression by nasogastric tube C) Periodic assessment for esophageal distension D) Monthly administration of injections of vitamin B12

Ans: D Feedback: Since vitamin B12 is absorbed in the stomach, the patient requires vitamin B12 replacement to prevent pernicious anemia. A gastrectomy precludes the use of a G tube. Since the stomach is absent, a nasogastric tube would not be indicated. As well, this is not possible in the home setting. Since there is no stomach to act as a reservoir and fluids and nutrients are passing directly into the jejunum, distension is unlikely.

4. A nurse is admitting a patient diagnosed with late-stage gastric cancer. The patient's family is distraught and angry that she was not diagnosed earlier in the course of her disease. What factor contributes to the fact that gastric cancer is often detected at a later stage? A) Gastric cancer does not cause signs or symptoms until metastasis has occurred. B) Adherence to screening recommendations for gastric cancer is exceptionally low. C) Early symptoms of gastric cancer are usually attributed to constipation. D) The early symptoms of gastric cancer are usually not alarming or highly unusual.

Ans: D Feedback: Symptoms of early gastric cancer, such as pain relieved by antacids, resemble those of benign ulcers and are seldom definitive. Symptoms are rarely a cause for alarm or for detailed diagnostic testing. Symptoms precede metastasis, however, and do not include constipation.

36. A nurse is caring for a patient who has a diagnosis of GI bleed. During shift assessment, the nurse finds the patient to betachycardic and hypotensive, and the patient has an episode of hematemesis while the nurse is in the room. In addition to monitoring the patient's vital signs and level of conscious, what would be a priority nursing action for this patient? A) Place the patient in a prone position. B) Provide the patient with ice water to slow any GI bleeding. C) Prepare for the insertion of an NG tube. D) Notify the physician.

Ans: D Feedback: The nurse must always be alert for any indicators of hemorrhagic gastritis, which include hematemesis (vomiting of blood), tachycardia, and hypotension. If these occur, the physician is notified and the patient's vital signs are monitored as the patient's condition warrants. Putting the patient in a prone position could lead to aspiration. Giving ice water is contraindicated as it would stimulate more vomiting.

12.A patient presents to the walk-in clinic complaining of vomiting and burning in her mid-epigastria. The nurse knows that in the process of confirming peptic ulcer disease, the physician is likely to order a diagnostic test to detect the presence of what? A)Infection with Helicobacter pylori B)Excessive stomach acid secretion C)An incompetent pyloric sphincter D)A metabolic acidbase imbalance

Ans: test bank A Feedback: H. pylori infection may be determined by endoscopy and histologic examination of a tissue specimen obtained by biopsy, or a rapid urease test of the biopsy specimen. Excessive stomach acid secretion leads to gastritis; however, peptic ulcers are caused by colonization of the stomach by H. pylori. Sphincter dysfunction and acidbase imbalances do not cause peptic ulcer disease.

3.A patient with a diagnosis of peptic ulcer disease has just been prescribed omeprazole (Prilosec). How should the nurse best describe this medication's therapeutic action? A)This medication will reduce the amount of acid secreted in your stomach. B)This medication will make the lining of your stomach more resistant to damage. C)This medication will specifically address the pain that accompanies peptic ulcer disease. D)This medication will help your stomach lining to repair itself.

Ans: test bank A Feedback: Proton pump inhibitors like Prilosec inhibit the synthesis of stomach acid. PPIs do not increase the durability of the stomach lining, relieve pain, or stimulate tissue repair.

11.A patient who experienced an upper GI bleed due to gastritis has had the bleeding controlled and the patient's condition is now stable. For the next several hours, the nurse caring for this patient should assess for what signs and symptoms of recurrence? A)Tachycardia, hypotension, and tachypnea B)Tarry, foul-smelling stools C)Diaphoresis and sudden onset of abdominal pain D)Sudden thirst, unrelieved by oral fluid administration

Ans: test bank A Feedback: Tachycardia, hypotension, and tachypnea are signs of recurrent bleeding. Patients who have had one GI bleed are at risk for recurrence. Tarry stools are expected short-term findings after a hemorrhage. Hemorrhage is not normally associated with sudden thirst or diaphoresis.

2.A patient comes to the clinic complaining of pain in the epigastric region. What assessment question during the health interview would most help the nurse determine if the patient has a peptic ulcer? A)Does your pain resolve when you have something to eat? B)Do over-the-counter pain medications help your pain? C)Does your pain get worse if you get up and do some exercise? D)Do you find that your pain is worse when you need to have a bowel movement?

Ans: test bank A Pain relief after eating is associated with duodenal ulcers. The pain of peptic ulcers is generally unrelated to activity or bowel function and may or may not respond to analgesics

28.Diagnostic imaging and physical assessment have revealed that a patient with peptic ulcer disease has suffered a perforated ulcer. The nurse recognizes that emergency interventions must be performed as soon as possible in order to prevent the development of what complication? A)Peritonitis B)Gastritis C)Gastroesophageal reflux D)Acute pancreatitis

Ans: test bank A Perforation is the erosion of the ulcer through the gastric serosa into the peritoneal cavity without warning. Chemical peritonitis develops within a few hours of perforation and is followed by bacterial peritonitis. Gastritis, reflux, and pancreatitis are not acute complications of a perforated ulcer.

39.A nurse is presenting a class at a bariatric clinic about the different types of surgical procedures offered by the clinic. When describing the implications of different types of surgeries, the nurse should address which of the following topics? Select all that apply. A)Specific lifestyle changes associated with each procedure B)Implications of each procedure for eating habits C)Effects of different surgeries on bowel function D)Effects of various bariatric surgeries on fertility E)Effects of different surgeries on safety of future immunizations

Ans: test bank A, B, C Different bariatric surgical procedures entail different lifestyle modifications; patients must be well informed about the specific lifestyle changes, eating habits, and bowel habits that may result from a particular procedure. Bariatric surgeries do not influence the future use of immunizations or fertility, though pregnancy should be avoided for 18 months after bariatric surgery.

27.A patient with a history of peptic ulcer disease has presented to the emergency department (ED) in distress. What assessment finding would lead the ED nurse to suspect that the patient has a perforated ulcer? A)The patient has abdominal bloating that developed rapidly. B)The patient has a rigid, boardlike abdomen that is tender. C)The patient is experiencing intense lower right quadrant pain. D)The patient is experiencing dizziness and confusion with no apparent hemodynamic changes.

Ans: test bank B Feedback: An extremely tender and rigid (boardlike) abdomen is suggestive of a perforated ulcer. None of the other listed signs and symptoms is suggestive of a perforated ulcer.

33.A patient who is obese is exploring bariatric surgery options and presented to a bariatric clinic for preliminary investigation. The nurse interviews the patient, analyzing and documenting the data. Which of the following nursing diagnoses may be a contraindication for bariatric surgery? A)Disturbed Body Image Related to Obesity B)Deficient Knowledge Related to Risks and Expectations of Surgery C)Anxiety Related to Surgery D)Chronic Low Self-Esteem Related to Obesity

Ans: test bank B Feedback: It is expected that patients seeking bariatric surgery may have challenges with body image and self-esteem related to their obesity. Anxiety is also expected when facing surgery. However, if the patient's knowledge remains deficient regarding the risks and realistic expectations for surgery, this may show that the patient is not an appropriate surgical candidate.

19.A patient comes to the bariatric clinic to obtain information about bariatric surgery. The nurse assesses the obese patient knowing that in addition to meeting the criterion of morbid obesity, a candidate for bariatric surgery must also demonstrate what? A)Knowledge of the causes of obesity and its associated risks B)Adequate understanding of required lifestyle changes C)Positive body image and high self-esteem D)Insight into why past weight loss efforts failed

Ans: test bank B Patients seeking bariatric surgery should be free of serious mental disorders and motivated to comply with lifestyle changes related to eating patterns, dietary choices, and elimination. While assessment of knowledge about causes of obesity and its associated risks as well as insight into the reasons why previous diets have been ineffective are included in the client's plan of care, these do not predict positive client outcomes following bariatric surgery. Most obese patients have an impaired body image and alteration in self-esteem. An obese patient with a positive body image would be unlikely to seek this surgery unless he or she was experiencing significant comorbidities.

5.A nurse is preparing to discharge a patient after recovery from gastric surgery. What is an appropriate discharge outcome for this patient? test bank A)The patient's bowel movements maintain a loose consistency. B)The patient is able to tolerate three large meals a day. C)The patient maintains or gains weight. D)The patient consumes a diet high in calcium.

Ans: test bank C Expected outcomes for the patient following gastric surgery include ensuring that the patient is maintaining or gaining weight (patient should be weighed daily), experiencing no excessive diarrhea, and tolerating six small meals a day. Patients may require vitamin B12 supplementation by the intramuscular route and do not require a diet excessively rich in calcium.

31.A patient who is obese has been unable to lose weight successfully using lifestyle modifications and has mentioned the possibility of using weight-loss medications. What should the nurse teach the patient about pharmacologic interventions for the treatment of obesity? A)Weight loss drugs have many side effects, and most doctors think they'll all be off the market in a few years. B)There used to be a lot of hope that medications would help people lose weight, but it's been shown to be mostly a placebo effect. C)Medications can be helpful, but few people achieve and maintain their desired weight loss with medications alone. D)Medications are rapidly become the preferred method of weight loss in people for whom diet and exercise have not worked.

Ans: test bank C Though antiobesity drugs help some patients lose weight, their use rarely results in loss of more than 10% of total body weight. Patients are consequently unlikely to attain their desired weight through medication alone. They are not predicted to disappear from the market and results are not attributed to a placebo effect.

32.A patient has been diagnosed with peptic ulcer disease and the nurse is reviewing his prescribed medication regimen with him. What is currently the most commonly used drug regimen for peptic ulcers? A)Bismuth salts, antivirals, and histamine-2 (H2) antagonists B)H2 antagonists, antibiotics, and bicarbonate salts C)Bicarbonate salts, antibiotics, and ZES D)Antibiotics, proton pump inhibitors, and bismuth salts

Ans: test bank D Feedback: Currently, the most commonly used therapy for peptic ulcers is a combination of antibiotics, proton pump inhibitors, and bismuth salts that suppress or eradicate H. pylori. H2 receptor antagonists are used to treat NSAID-induced ulcers and other ulcers not associated with H. pylori infection, but they are not the drug of choice. Bicarbonate salts are not used. ZES is the Zollinger-Ellison syndrome and not a drug.

13.A patient with a peptic ulcer disease has had metronidazole (Flagyl) added to his current medication regimen. What health education related to this medication should the nurse provide? A)Take the medication on an empty stomach. B)Take up to one extra dose per day if stomach pain persists. C)Take at bedtime to mitigate the effects of drowsiness. D)Avoid drinking alcohol while taking the drug.

Ans: test bank D Alcohol must be avoided when taking Flagyl and the medication should be taken with food. This drug does not cause drowsiness and the dose should not be adjusted by the patient.

1.A nurse is caring for a patient who just has been diagnosed with a peptic ulcer. When teaching the patient about his new diagnosis, how should the nurse best describe a peptic ulcer? A)Inflammation of the lining of the stomach B)Erosion of the lining of the stomach or intestine C)Bleeding from the mucosa in the stomach D)Viral invasion of the stomach wall

Ans:B test bank A peptic ulcer is erosion of the lining of the stomach or intestine. Peptic ulcers are often accompanied by bleeding and inflammation, but these are not the definitive characteristics.

14. A patient was treated in the emergency department and critical care unit after ingesting bleach. What possible complication of the resulting gastritis should the nurse recognize? A) Esophageal or pyloric obstruction related to scarring B) Uncontrolled proliferation of H. pylori C) Gastric hyperacidity related to excessive gastrin secretion D) Chronic referred pain in the lower abdomen

Ans:test bank A Feedback: A severe form of acute gastritis is caused by the ingestion of strong acid or alkali, which may cause the mucosa to become gangrenous or to perforate. Scarring can occur, resulting in pyloric stenosis (narrowing or tightening) or obstruction. Chronic referred pain to the lower abdomen is a symptom of peptic ulcer disease, but would not be an expected finding for a patient who has ingested a corrosive substance. Bacterial proliferation and hyperacidity would not occur.

16. A patient is one month postoperative following restrictive bariatric surgery. The patient tells the clinic nurse that he has been having trouble swallowing for the past few days. What recommendation should the nurse make? A) Eating more slowly and chewing food more thoroughly B) Taking an OTC antacid or drinking a glass of milk prior to each meal C) Chewing gum to cause relaxation of the lower esophageal sphincter D) Drinking at least 12 ounces of liquid with each meal

Ans:test bank A Feedback: Dysphagia may be prevented by educating patients to eat slowly, to chew food thoroughly, and to avoid eating tough foods such as steak or dry chicken or doughy bread. After bariatric procedures, patients should normally not drink beverages with meals. Medications or chewing gum will not alleviate this problem.

26. A patient has been admitted to the hospital after diagnostic imaging revealed the presence of a gastric outlet obstruction (GOO). What is the nurse's priority intervention? A) Administration of antiemetics B) Insertion of an NG tube for decompression C) Infusion of hypotonic IV solution D) Administration of proton pump inhibitors as ordered

Ans:test bank B Feedback: In treating the patient with gastric outlet obstruction, the first consideration is to insert an NG tube to decompress the stomach. This is a priority over fluid or medication administration.

6.A nurse caring for a patient who has had bariatric surgery is developing a teaching plan in anticipation of the patient's discharge. Which of the following is essential to include? A)Drink a minimum of 12 ounces of fluid with each meal. B)Eat several small meals daily spaced at equal intervals. C)Choose foods that are high in simple carbohydrates. D)Sit upright when eating and for 30 minutes afterward.

Ans:test bank B Due to decreased stomach capacity, the patient must consume small meals at intervals to meet nutritional requirements while avoiding a feeling of fullness and complications such as dumping syndrome. The patient should not consume fluids with meals and low-Fowler's positioning is recommended during and after meals. Carbohydrates should be limited.

10.A nurse is assessing a patient who has peptic ulcer disease. The patient requests more information about the typical causes of Helicobacter pylori infection. What would it be appropriate for the nurse to instruct the patient? test bank A)Most affected patients acquired the infection during international travel. B)Infection typically occurs due to ingestion of contaminated food and water. C)Many people possess genetic factors causing a predisposition to H. pylori infection. D)The H. pylori microorganism is endemic in warm, moist climates.

Ans:test bank B Most peptic ulcers result from infection with the gram-negative bacteria H. pylori, which may be acquired through ingestion of food and water. The organism is endemic to all areas of the United States. Genetic factors have not been identified.

23. A nurse is providing care for a patient who is postoperative day 2 following gastric surgery. The nurse's assessment should be planned in light of the possibility of what potential complications? Select all that apply. A) Malignant hyperthermia B) Atelectasis C) Pneumonia D) Metabolic imbalances E) Chronic gastritis

Ans:test bank B, C, D Feedback: After surgery, the nurse assesses the patient for complications secondary to the surgical intervention, such as pneumonia, atelectasis, or metabolic imbalances resulting from the GI disruption. Malignant hyperthermia is an intraoperative complication. Chronic gastritis is not a surgical complication.

17. A patient is receiving education about his upcoming Billroth I procedure (gastroduodenostomy). This patient should be informed that he may experience which of the following adverse effects associated with this procedure? A) Persistent feelings of hunger and thirst B) Constipation or bowel incontinence C) Diarrhea and feelings of fullness D) Gastric reflux and belching

Ans:test bank C Feedback: Following a Billroth I, the patient may have problems with feelings of fullness, dumping syndrome, and diarrhea. Hunger and thirst, constipation, and gastric reflux are not adverse effects associated with this procedure.

20. A nurse is providing patient education for a patient with peptic ulcer disease secondary to chronic nonsteroidal anti-inflammatory drug (NSAID) use. The patient has recently been prescribed misoprostol (Cytotec). What would the nurse be most accurate in informing the patient about the drug? A) It reduces the stomach's volume of hydrochloric acid B) It increases the speed of gastric emptying C) It protects the stomach's lining D) It increases lower esophageal sphincter pressure

Ans:test bank C Feedback: Misoprostol is a synthetic prostaglandin that, like prostaglandin, protects the gastric mucosa. NSAIDs decrease prostaglandin production and predispose the patient to peptic ulceration. Misoprostol does not reduce gastric acidity, improve emptying of the stomach, or increase lower esophageal sphincter pressure.

7.A nurse is completing a health history on a patient whose diagnosis is chronic gastritis. Which of the data should the nurse consider most significantly related to the etiology of the patient's health problem? test bank A)Consumes one or more protein drinks daily. B)Takes over-the-counter antacids frequently throughout the day. C)Smokes one pack of cigarettes daily. D)Reports a history of social drinking on a weekly basis.

Ans:test bank C Feedback: Nicotine reduces secretion of pancreatic bicarbonate, which inhibits neutralization of gastric acid and can underlie gastritis. Protein drinks do not result in gastric inflammation. Antacid use is a response to experiencing symptoms of gastritis, not the etiology of gastritis. Alcohol ingestion can lead to gastritis; however, this generally occurs in patients with a history of consumption of alcohol on a daily basis.

15. A patient who underwent gastric banding 3 days ago is having her diet progressed on a daily basis. Following her latest meal, the patient complains of dizziness and palpitations. Inspection reveals that the patient is diaphoretic. What is the nurse's best action? A) Insert a nasogastric tube promptly. B) Reposition the patient supine. C) Monitor the patient closely for further signs of dumping syndrome. D) Assess the patient for signs and symptoms of aspiration.

Ans:test bank C Feedback: The patient's symptoms are characteristic of dumping syndrome, which results in a sensation of fullness, weakness, faintness, dizziness, palpitations, diaphoresis, cramping pains, and diarrhea. Aspiration is a less likely cause for the patient's symptoms. Supine positioning will likely exacerbate the symptoms and insertion of an NG tube is contraindicated due to the nature of the patient's surgery.

24. A patient is undergoing diagnostic testing for a tumor of the small intestine. What are the most likely symptoms that prompted the patient to first seek care? A) Hematemesis and persistent sensation of fullness B) Abdominal bloating and recurrent constipation C) Intermittent pain and bloody stool D) Unexplained bowel incontinence and fatty stools

Ans:test bank C Feedback: When the patient is symptomatic from a tumor of the small intestine, benign tumors often present with intermittent pain. The next most common presentation is occult bleeding. The other listed signs and symptoms are not normally associated with the presentation of small intestinal tumors.

22. A patient has just been diagnosed with acute gastritis after presenting in distress to the emergency department with abdominal symptoms. What would be the nursing care most needed by the patient at this time? test bank A)Teaching the patient about necessary nutritional modification B)Helping the patient weigh treatment options C)Teaching the patient about the etiology of gastritis D)Providing the patient with physical and emotional support

Ans:test bank D Feedback: For acute gastritis, the nurse provides physical and emotional support and helps the patient manage the symptoms, which may include nausea, vomiting, heartburn, and fatigue. The scenario describes a newly diagnosed patient; teaching about the etiology of the disease, lifestyle modifications, or various treatment options would be best provided at a later time.

18. test bank A patient has experienced symptoms of dumping syndrome following bariatric surgery. To what physiologic phenomenon does the nurse attribute this syndrome? A)Irritation of the phrenic nerve due to diaphragmatic pressure B)Chronic malabsorption of iron and vitamins A and C C)Reflux of bile into the distal esophagus D)A sudden release of peptides

Ans:test bank D Feedback: For many years, it had been theorized that the hypertonic gastric food boluses that quickly transit into the intestines drew extracellular fluid from the circulating blood volume into the small intestines to dilute the high concentration of electrolytes and sugars, resulting in symptoms. Now, it is thought that this rapid transit of the food bolus from the stomach into the small intestines instead causes a rapid and exuberant release of metabolic peptides that are responsible for the symptoms of dumping syndrome. It is not a result of phrenic nerve irritation, malabsorption, or bile reflux.

25. A patient is recovering in the hospital following gastrectomy. The nurse notes that the patient has become increasingly difficult to engage and has had several angry outbursts at various staff members in recent days. The nurse's attempts at therapeutic dialogue have been rebuffed. What is the nurse's most appropriate action? A) Ask the patient's primary care provider to liaise between the nurse and the patient. B) Delegate care of the patient to a colleague. C) Limit contact with the patient in order to provide privacy. D) Make appropriate referrals to services that provide psychosocial support.

Ans:test bank D Feedback: The nurse should enlist the services of clergy, psychiatric clinical nurse specialists, psychologists, social workers, and psychiatrists, if needed. This is preferable to delegating care, since the patient has become angry with other care providers as well. It is impractical and inappropriate to expect the primary care provider to act as a liaison. It would be inappropriate and unsafe to simply limit contact with the patient.

A patient has been diagnosed with peptic ulcer disease and the nurse is reviewing his prescribed medication regimen with him. What is currently the most commonly used drug regimen for peptic ulcers? a) Bicarbonate salts, antibiotics, and ZES b) Antibiotics, proton pump inhibitors, and bismuth salts c) H2 antagonists, antibiotics, and bicarbonate salts d) Bismuth salts, antivirals, and histamine-2 (H2) antagonists

Antibiotics, proton pump inhibitors, and bismuth salts Correct Explanation: Currently, the most commonly used therapy for peptic ulcers is a combination of antibiotics, proton pump inhibitors, and bismuth salts that suppress or eradicate H. pylori. H2 receptor antagonists are used to treat NSAID-induced ulcers and other ulcers not associated with H. pylori infection, but they are not the drug of choice. Bicarbonate salts are not used. ZES is the Zollinger-Ellison syndrome and not a drug. (less) - Page 1267

A client with gastric cancer is scheduled to undergo a Billroth II procedure. The client's spouse asks how much of the client's stomach will be removed. Which of the following would be the most accurate response from the nurse? a) The amount will depend on the client's weight. b) Approximately 75% c) Approximately 50% d) Approximately 25%

Approximately 75% Explanation: The Billroth II is a wide resection that involves removing approximately 75% of the stomach and decreases the possibility of lymph node spread or metastatic recurrence. Page 1279

When caring for a client with an acute exacerbation of a peptic ulcer, the nurse finds the client doubled up in bed with severe pain to his right shoulder. The intial appropriate action by the nurse is to a) Assess the client's abdomen and vital signs. b) Irrigate the client's NG tube. c) Place the client in the high-Fowler's position. d) Notify the health care provider.

Assess the client's abdomen and vital signs. Correct Explanation: Signs and symptoms of perforation includes sudden, severe upper abdominal pain (persisting and increasing in intensity); pain may be referred to the shoulders, especially the right shoulder, because of irritation of the phrenic nerve in the diaphragm. The nurse should assess the vital signs and abdomen prior to notifying the physician. Irrigation of the NG tube should not be performed because the additional fluid may be spilled into the peritoneal cavity, and the client should be placed in a position of comfort, usually on the side with the head slightly elevated. Page 1271

The nurse is assessing a client with a bleeding gastric ulcer. When examining the client's stool, which of the following characteristics would the nurse be most likely to find? a) Green color and texture b) Clay-like quality c) Black and tarry appearance d) Bright red blood in stool

Black and tarry appearance Correct Explanation: Black and tarry stools (melena) are a sign of bleeding in the upper gastrointestinal (GI) tract. As the blood moves through the GI system, digestive enzymes turn red blood to black. Bright red blood in the stool is a sign of lower GI bleeding. Green color and texture is a distractor for this question. Clay-like stools are a characteristic of biliary disorders. Page 1262

The nurse is conducting an admission assessment and determines that the patient's BMI is 37. The nurse documents the BMI as being which of the following classes of obesity? a) Class II b) Overweight c) Class III d) Class I

Class II Correct Explanation: The nurse documents the BMI of 37 as class II obesity (BMI 35 to 40).The other classifications are as follows: overweight (BMI 25 to 30); class I (BMI 30 to 35); class III (BMI > 40). - Page 1272

A morbidly obese client asks the nurse if medications are available to assist with weight loss. The nurse knows that the client would not be a candidate for sibutramine HCl (Meridia) if the following is part of the client's health history: a) Use of lithium b) Coronary artery disease c) Diabetes d) Peptic ulcer disease

Coronary artery disease Explanation: Sibutramine HCl, which requires a prescription, decreases appetite by inhibiting the reuptake of serotonin and norepinephrine. It may increase blood pressure and should not be taken by people with a history of coronary artery disease, angina pectoris, dysrhythmias, or kidney disease; by those taking antidepressants or monoamine oxidase inhibitors; or by pregnant or nursing women. Page 1273

The nurse is assessing an 80-year-old patient for signs and symptoms of gastric cancer. The nurse differentiates which of the following to be a sign/symptom of gastric cancer in the geriatric patient, but not in a patient under the age of 75? a) Ascites b) Hepatomegalia c) Abdominal mass d) Agitation

Correct response: Agitation Explanation: The nurse differentiates that agitation, along with confusion and restlessness, may be the only signs/symptoms seen of gastric cancer in the older patient. Abdominal mass, hepatomegaly, and ascites may all be signs/symptoms of advanced gastric cancer. - Page 1279

Symptoms associated with pyloric obstruction include all of the following except: a) Epigastric fullness b) Anorexia c) Nausea and vomiting d) Diarrhea

Correct response: Diarrhea Explanation: Pyloric obstruction, also called gastric outlet obstruction (GOO), occurs when the area distal to the pyloric sphincter becomes scarred and stenosed from spasm or edema or from scar tissue that forms when an ulcer alternately heals and breaks down. The client may have nausea and vomiting, constipation, epigastric fullness, anorexia, and, later, weight loss. (less)

The nurse in the ED is admitting a patient with bloody stools. The nurse documents this finding as being which of the following? a) Steatorrhea b) Tarry stools c) Melena d) Hematochezia

Correct response: Hematochezia Explanation: The nurse should document the finding of bloody stools as hematochezia. Melena is the term used for tarry black stools with occult blood. Steatorrhea is the term utilized for fatty stools that have an oily appearance and float in water. (less) Page 1262

A client experienced extensive burns and 72 hours later has developed an ulcer. Which of the following types of ulcer is most likely in this client? a) Hashimoto's ulcer b) Cushing's ulcer c) Stress ulcer d) Curling's ulcer

Curling's ulcer Explanation: Curling's ulcer is frequently observed about 72 hours after extensive burns and involves the antrum of the stomach or duodenum. Page 1266

Which of the following ulcers is associated with extensive burn injury? a) Peptic ulcer b) Duodenal ulcer c) Cushing's ulcer d) Curling's ulcer

Curling's ulcer Explanation: Curling's ulcer is frequently observed about 72 hours after extensive burns and involves the antrum of the stomach or the duodenum. Page 1266

Which of the following medications used for the treatment of obesity prevents the reuptake of serotonin and norepinephrine? a) Orlistat (Xenical) b) Fluoxetine hydrochloride (Prozac) c) Bupropion hydrochloride (Wellbutrin) d) Lorcaserin (Belviq)

D Explanation: Lorcaserin (Belviq)) inhibits the reuptake of serotonin and norepinephrine. Meridia decreases appetite. Orlistat (Xenical) prevents the absorption of triglycerides. Side effects of Xenical may include increased bowel movements, gas with oily discharge, decreased food absorption, decreased bile flow, and decreased absorption of some vitamins. Bupropion hydrochloride (Wellbutrin) is an antidepressant medication. Fluoxetine hydrochloride (Prozac) has not been approved by the FDA for use in the treatment of obesity. Page 1273

A patient who is obese is exploring bariatric surgery options and presented to a bariatric clinic for preliminary investigation. The nurse interviews the patient, analyzing and documenting the data. Which of the following nursing diagnoses may be a contraindication for bariatric surgery? a) Disturbed Body Image Related to Obesity b) Deficient Knowledge Related to Risks and Expectations of Surgery c) Anxiety Related to Surgery d) Chronic Low Self-Esteem Related to Obesity

Deficient Knowledge Related to Risks and Expectations of Surgery Explanation: It is expected that patients seeking bariatric surgery may have challenges with body image and self-esteem related to their obesity. Anxiety is also expected when facing surgery. However, if the patient's knowledge remains deficient regarding the risks and realistic expectations for surgery, this may show that the patient is not an appropriate surgical candidate. Page 1273

Symptoms associated with pyloric obstruction include all of the following except: a) Anorexia b) Nausea and vomiting c) Epigastric fullness d) Diarrhea

Diarrhea Explanation: Pyloric obstruction, also called gastric outlet obstruction (GOO), occurs when the area distal to the pyloric sphincter becomes scarred and stenosed from spasm or edema or from scar tissue that forms when an ulcer alternately heals and breaks down. The client may have nausea and vomiting, constipation, epigastric fullness, anorexia, and, later, weight loss. (less)

Which of the following appears to be a significant factor in the development of gastric cancer? a) Gender b) Diet c) Age d) Ethnicity

Diet Correct Explanation: Diet seems to be a significant factor: a diet high in smoked, salted, or pickled foods and low in fruits and vegetables may increase the risk of gastric cancer. The typical patient with gastric cancer is between 50 and 70 years of age. Men have a higher incidence than women. Native Americans, Hispanic Americans, and African Americans are twice as likely as Caucasian Americans to develop gastric cancer. Page 1278

Clients with Type O blood are at higher risk for which of the following GI disorders? a) Duodenal ulcers b) Gastric cancer c) Esophageal varices d) Diverticulitis

Duodenal ulcers Correct Explanation: Familial tendency also may be a significant predisposing factor. People with blood type O are more susceptible to peptic ulcers than are those with blood type A, B, or AB. Blood type is not a predisposing factor for gastric cancer, esophageal varices, and diverticulitis. (less) Page 1266

Which of the following is the first portion of the small intestine? a) Duodenum b) Peritoneum c) Pylorus d) Omentum

Duodenum Correct Explanation: The duodenum is the first portion of the small intestine, between the stomach and the jejunum. The pylorus is the opening between the stomach and duodenum. The peritoneum is the thin membrane that lines the inside of the abdominal wall and covers all the abdominal organs. The omentum is the fold of the peritoneum that surrounds the stomach and other organs of the abdomen. - Page 1265

The nursing student approaches his instructor to discuss the plan of care for his client diagnosed with peptic ulcer disease. The student asks what is the most common site for peptic ulcer formation? The instructor would state which one of the following? a) Stomach b) Pylorus c) Duodenum d) Esophagus

Duodenum Explanation: Peptic ulcers occur mainly in the gastroduodenal mucosa because this tissue cannot withstand the digestive action of gastric acid (HCl) and pepsin.

A patient has received a diagnosis of gastric cancer and is awaiting a surgical date. During the preoperative period, the patient should adopt what dietary guidelines? a) Try to maintain the pre-diagnosis pattern of eating. b) Eat frequent, low-fat meals with high protein content. c) Eat frequent meals with an equal balance of fat, carbohydrates, and protein. d) Eat small, frequent meals with high calorie and vitamin content.

Eat small, frequent meals with high calorie and vitamin content. Explanation: The nurse encourages the patient to eat small, frequent portions of nonirritating foods to decrease gastric irritation. Food supplements should be high in calories, as well as vitamins A and C and iron, to enhance tissue repair. Page 1280

A physician suspects that a client has peptic ulcer disease. With which of the following diagnostic procedures would the nurse most likely prepare to assist? a) Barium study of the upper gastrointestinal tract b) Endoscopy c) Stool antigen test d) Gastric secretion study

Endoscopy Explanation: Barium study of the upper GI tract may show an ulcer; however, endoscopy is the preferred diagnostic procedure because it allows direct visualization of inflammatory changes, ulcers, and lesions. Through endoscopy, a biopsy of the gastric mucosa and of any suspicious lesions can be obtained. Endoscopy may reveal lesions that, because of their size or location, are not evident on x-ray studies. Less invasive diagnostic measures for detecting H. pylori include serologic testing for antibodies against the H. pylori antigen, stool antigen test, and urea breath test. Page 1267

A nurse is performing the admission assessment of a patient whose high body mass index (BMI) corresponds to class III obesity. In order to ensure empathic and patient-centered care, the nurse should do which of the following? a) Dialogue with the patient about the lifestyle and psychosocial factors that resulted in obesity. b) Examine one's own attitudes towards obesity in general and the patient in particular. c) Elicit the patient's short-term and long-term goals for weight loss. d) Describe one's own struggles with weight gain and weight loss to the patient.

Examine one's own attitudes towards obesity in general and the patient in particular. Explanation: Studies suggest that health care providers, including nurses, harbor negative attitudes towards obese patients. Nurses have a responsibility to examine these attitudes and change them accordingly. This is foundational to all other areas of assessing this patient. Page 1272

A nurse is providing anticipator guidance to a patient who is preparing for bariatric surgery. The nurse learns that the patient is anxious about numerous aspects of the surgery. What intervention is most appropriate to alleviate the patient's anxiety? a) Obtain an order for a PRN benzodiazepine. b) Facilitate the patient's contact with a support group. c) Encourage the patient to focus on the benefits of the surgery. d) Emphasize the fact that bariatric surgery has a low risk of complications.

Facilitate the patient's contact with a support group. Explanation: Support groups can be highly beneficial in relieving preoperative and postoperative anxiety and in promoting healthy coping. This is preferable to antianxiety medications. Downplaying the risks of surgery or focusing solely on the benefits is a simplistic and patronizing approach. Page 1268

A patient has been prescribed orlistat (Xenical) for the treatment of obesity. When providing relevant health education for this patient, the nurse should ensure the patient is aware of what potential adverse effect of treatment? a) Abdominal pain b) Flatus with oily discharge c) Heat intolerance d) Bowel incontinence

Flatus with oily discharge Explanation: Side effects of orlistat include increased frequency of bowel movements, gas with oily discharge, decreased food absorption, decreased bile flow, and decreased absorption of some vitamins. This drug does not cause bowel incontinence, abdominal pain, or heat intolerance. Page 1272

A client is recovering from gastric surgery. Which of the following is the correct position for the nurse to place this client? a) Supine b) Semi-Fowler's c) Trendlenberg d) Fowler's

Fowler's Correct Explanation: Placing the client in the Fowler's position after gastric surgery promotes comfort and allows emptying of the stomach. Page 1276

Which of the following surgical procedures for obesity utilizes a prosthetic device to restrict oral intake? a) Biliopancreatic diversion with duodenal switch b) Vertical-banded gastroplasty c) Roux-en-Y gastric bypass d) Gastric banding

Gastric banding pg 1275 In gastric banding, a prosthetic device is used to restrict oral intake by creating a small pouch of 10 to 15 milliliters that empties through the narrow outlet into the remainder of the stomach. Roux-en-Y gastric bypass uses a horizontal row of staples across the fundus of the stomach to create a pouch with a capacity of 20 to 30 mL. Vertical-banded gastroplasty involves placement of a vertical row of staples along the lesser curvature of the stomach, creating a new, small gastric pouch. Biliopancreatic diversion with duodenal switch combines gastric restriction with intestinal malabsorption.

The nurse is assessing a patient with an ulcer for signs and symptoms of hemorrhage. The nurse interprets which of the following as a sign/symptom of possible hemorrhage? a) Hematemesis b) Polyuria c) Bradycardia d) Hypertension

Hematemesis Correct Explanation: The nurse interprets hematemesis as a sign/symptom of possible hemorrhage from the ulcer. Other signs that can indicate hemorrhage include tachycardia, hypotension, and oliguria/anuria. (less) - Page 1270

The nurse in the ED is admitting a patient with bloody stools. The nurse documents this finding as being which of the following? a) Melena b) Steatorrhea c) Tarry stools d) Hematochezia

Hematochezia Explanation: The nurse should document the finding of bloody stools as hematochezia. Melena is the term used for tarry black stools with occult blood. Steatorrhea is the term utilized for fatty stools that have an oily appearance and float in water. (less) Page 1262

Which of the following is a true statement regarding the nursing considerations in administration of Metronidazole (Flagyl)? a) Flagyl decreases the effect of Coumadin. b) It leaves a metallic taste in the mouth. c) It may cause weight gain. d) The drug should be given prior to meals.

It leaves a metallic taste in the mouth. Explanation: Flagyl leaves a metallic taste in the mouth. It may cause anorexia and should be given with meals to decrease GI upset. Flagyl increases the blood-thinning effects of warfarin (Coumadin). Page 1264

A client with severe peptic ulcer disease has undergone surgery and is several hours postoperative. During assessment, the nurse notes that the client has developed cool skin, tachycardia, and labored breathing; the client also appears to be confused. Which of the following complications has the client most likely developed? a) Penetration b) Hemorrhage c) Perforation d) Pyloric obstruction

Hemorrhage Correct Explanation: Signs of hemorrhage following surgery include cool skin, confusion, increased heart rate, labored breathing, and blood in the stool. Signs of penetration and perforation are severe abdominal pain, rigid and tender abdomen, vomiting, elevated temperature, and increased heart rate. Indicators of pyloric obstruction are nausea, vomiting, distended abdomen, and abdominal pain. Page 1270

A client with severe peptic ulcer disease has undergone surgery and is several hours postoperative. During assessment, the nurse notes that the client has developed cool skin, tachycardia, and labored breathing; the client also appears to be confused. Which of the following complications has the client most likely developed? a) Hemorrhage b) Perforation c) Penetration d) Pyloric obstruction

Hemorrhage Correct Explanation: Signs of hemorrhage following surgery include cool skin, confusion, increased heart rate, labored breathing, and blood in the stool. Signs of penetration and perforation are severe abdominal pain, rigid and tender abdomen, vomiting, elevated temperature, and increased heart rate. Indicators of pyloric obstruction are nausea, vomiting, distended abdomen, and abdominal pain. Page 1270

The nurse is cautiously assessing a client admitted with peptic ulcer disease because the most common complication that occurs in 10% to 20% of clients is: a) Hemorrhage b) Pyloric obstruction c) Intractable ulcer d) Perforation

Hemorrhage pg 1270 Hemorrhage, the most common complication, occurs in 10% to 20% of clients with peptic ulcers. Bleeding may be manifested by hematemesis or melena. Perforation is erosion of the ulcer through the gastric serosa into the peritoneal cavity without warning. Intractable ulcer refers to one that is hard to treat, relieve, or cure. Pyloric obstruction, also called gastric outlet obstruction (GOO), occurs when the area distal to the pyloric sphincter becomes scarred and stenosed from spasm or edema or from scar tissue that forms when an ulcer alternately heals and breaks down.

A patient who had bariatric surgery complains of diarrhea. The nurse recommends which of the following dietary changes? a) Increased fiber b) Decreased carbohydrates c) Increased protein d) Decreased fat

Increased fiber Explanation: The nurse recommends that the patient increase fiber in the diet because a high-fiber diet can decrease both diarrhea and constipation after bariatric surgery. Page 1277

The nurse is reviewing the medication record of a client with acute gastritis. Which medication, if noted on the client's record, would the nurse question? a) Indomethacin (Indocin) b) Propranolol hydrolchloride (Inderal) c) Furosemide (Lasix) d) Digoxin (Lanoxin)

Indomethacin (Indocin) Explanation: Indomethacin is a nonsterioidal anti-inflammatory drug and can cause ulceration of the esophagus, stomach, or small intestine. Indomethacin is contraindicated in a client with a gastrointestinal disorder. The other medications are not contraindicated in clients with gastrointestinal disorders. (less) Page 1047

The client has been taking famotidine (Pepcid) at home. The nurse prepares a teaching plan for the client indicating that the medication acts primarily to achieve which of the following? a) Improve the mixing of foods and gastric secretions. b) Neutralize acid in the stomach. c) Shorten the time required for digestion in the stomach. d) Inhibit gastric acid secretions.

Inhibit gastric acid secretions. Correct Explanation: Famotidine is useful for treating and preventing ulcers and managing gastroesophageal reflux disease. It functions by inhibiting the action of histamine at the H-2 receptor site located in the gastric parietal cells, thus inhibiting gastric acid secretion. (less) - Page 1264

A patient has been admitted to the hospital after diagnostic imaging revealed the presence of a gastric outlet obstruction (GOO). What is the nurse's priority intervention? a) Insertion of an NG tube for decompression b) Administration of proton pump inhibitors as ordered c) Administration of antiemetics d) Infusion of hypotonic IV solution

Insertion of an NG tube for decompression Correct Explanation: In treating the patient with gastric outlet obstruction, the first consideration is to insert an NG tube to decompress the stomach. This is a priority over fluid or medication administration. Page 1271

A patient is undergoing diagnostic testing for a tumor of the small intestine. What are the most likely symptoms that prompted the patient to first seek care? a) Unexplained bowel incontinence and fatty stools b) Abdominal bloating and recurrent constipation c) Hematemesis and persistent sensation of fullness d) Intermittent pain and bloody stool

Intermittent pain and bloody stool Correct Explanation: When the patient is symptomatic from a tumor of the small intestine, benign tumors often present with intermittent pain. The next most common presentation is occult bleeding. The other listed signs and symptoms are not normally associated with the presentation of small intestinal tumors. Page 1281

The nurse is caring for a client who underwent a subtotal gastrectomy 36 hours ago. The client has a nasogastric (NG) tube. The nurse knows to do the following with the NG: a) Clamp the NG tube if the client complains of nausea. b) Apply suction to the NG tube every hour. c) Reposition the NG tube if pulled out. d) Irrigate the NG tube gently with normal saline if ordered.

Irrigate the NG tube gently with normal saline if ordered. Correct Explanation: The nurse maintains functioning of the NG tube to prevent distention and secures the tube to prevent dislocation, which may result in increased pain and tension. The nurse can gently irrigate the tube if ordered, but must be careful not to reposition it. Repositioning can cause bleeding. Suction should be applied continuously---not every hour. The NG tube shouldn't be clamped postoperatively because secretions and gas will accumulate, stressing the suture line. Page 1271

Which of the following is a true statement regarding the nursing considerations in administration of metronidazole (Flagyl)? a) It leaves a metallic taste in the mouth. b) Flagyl decreases the effect of Coumadin. c) It may cause weight gain. d) The drug should be given prior to meals.

It leaves a metallic taste in the mouth. Explanation: Flagyl leaves a metallic taste in the mouth. It may cause anorexia and should be given with meals to decrease GI upset. Flagyl increases the blood-thinning effects of warfarin (Coumadin). Page 1264

A nurse is providing patient education for a patient with peptic ulcer disease secondary to chronic nonsteroidal anti-inflammatory drug (NSAID) use. The patient has recently been prescribed misoprostol (Cytotec). What would the nurse be most accurate in informing the patient about the drug? a) It increases the speed of gastric emptying b) It increases lower esophageal sphincter pressure c) It protects the stomach's lining d) It reduces the stomach's volume of hydrochloric acid

It protects the stomach's lining Correct Explanation: Misoprostol is a synthetic prostaglandin that, like prostaglandin, protects the gastric mucosa. NSAIDs decrease prostaglandin production and predispose the patient to peptic ulceration. Misoprostol does not reduce gastric acidity, improve emptying of the stomach, or increase lower esophageal sphincter pressure. Page 1264

A patient is recovering in the hospital following gastrectomy. The nurse notes that the patient has become increasingly difficult to engage and has had several angry outbursts at various staff members in recent days. The nurse's attempts at therapeutic dialogue have been rebuffed. What is the nurse's most appropriate action? a) Make appropriate referrals to services that provide psychosocial support. b) Ask the patient's primary care provider to liaise between the nurse and the patient. c) Delegate care of the patient to a colleague. d) Limit contact with the patient in order to provide privacy.

Make appropriate referrals to services that provide psychosocial support. Correct Explanation: The nurse should enlist the services of clergy, psychiatric clinical nurse specialists, psychologists, social workers, and psychiatrists, if needed. This is preferable to delegating care, since the patient has become angry with other care providers as well. It is impractical and inappropriate to expect the primary care provider to act as a liaison. It would be inappropriate and unsafe to simply limit contact with the patient. Page 1280

A patient is recovering in the hospital following gastrectomy. The nurse notes that the patient has become increasingly difficult to engage and has had several angry outbursts at various staff members in recent days. The nurse's attempts at therapeutic dialogue have been rebuffed. What is the nurse's most appropriate action? a) Ask the patient's primary care provider to liaise between the nurse and the patient. b) Limit contact with the patient in order to provide privacy. c) Make appropriate referrals to services that provide psychosocial support. d) Delegate care of the patient to a colleague.

Make appropriate referrals to services that provide psychosocial support. Explanation: The nurse should enlist the services of clergy, psychiatric clinical nurse specialists, psychologists, social workers, and psychiatrists, if needed. This is preferable to delegating care, since the patient has become angry with other care providers as well. It is impractical and inappropriate to expect the primary care provider to act as a liaison. It would be inappropriate and unsafe to simply limit contact with the patient. Page 1280

Rebleeding may occur from a peptic ulcer and often warrants surgical interventions. Signs of bleeding include which of the following? a) Bradypnea b) Hypertension c) Mental confusion d) Bradycardia

Mental confusion Correct Explanation: Signs of bleeding include tachycardia, tachypnea, hypotension, mental confusion, thirst, and oliguria. - Page 1271

Which of the following is a true statement regarding gastric cancer? a) The prognosis for gastric cancer is good. b) Women have a higher incidence of gastric cancer. c) Most cases are discovered prior to metastasis. d) Most patients are asymptomatic during the early stage of the disease.

Most patients are asymptomatic during the early stage of the disease. Correct Explanation: Most patients are asymptomatic during the early stage of the disease. Men have a higher incidence of gastric cancer. The prognosis is poor because the diagnosis is usually made late because most patients are asymptomatic during the early stage. Most cases of gastric cancer are discovered only after local invasion has advanced or metastases are present. Page 1265

Which of the following is a true statement regarding gastric cancer? a) The prognosis for gastric cancer is good. b) Women have a higher incidence of gastric cancer. c) Most patients are asymptomatic during the early stage of the disease. d) Most cases are discovered prior to metastasis.

Most patients are asymptomatic during the early stage of the disease. Explanation: Most patients are asymptomatic during the early stage of the disease. Men have a higher incidence of gastric cancer than women. The prognosis is poor because the diagnosis is usually made late because most patients are asymptomatic during the early stage. Most cases of gastric cancer are discovered only after local invasion has advanced or metastases are present. Page 1278

The nurse is caring for a patient after bariatric surgery who experiences symptoms of gastric outlet obstruction. Which of the following are contraindicated? a) Surgical revision b) NG tube c) Balloon dilation d) Endoscopic procedure

NG tube Explanation: NG tube insertion is contraindicated in patients who have had bariatric surgery. Alternative treatment options include endoscopic procedures, balloon dilation, and/or surgical revision. (less) - Page 1277

The nurse in the ED admits a patient with suspected gastric outlet obstruction. The patient's symptoms include nausea and vomiting. The nurse anticipates that the physician will issue which of the following orders? a) Nasogastric (NG) tube insertion b) Oral contrast c) Stool specimen d) Pelvic x-ray

Nasogastric (NG) tube insertion Explanation: The nurse anticipates an order for NG tube insertion to decompress the stomach. Pelvic x-ray, oral contrast, and stool specimens are not indicated at this time. Page 1271

A nursing student notes that a client has a new physician order for a proton-pump inhibitor. The student knows that a frequently prescribed proton-pump inhibitor of gastric acid is a) Nexium b) Zantac c) Pepcid d) Tagamet

Nexium Explanation: Nexium decreases gastric acid secretion by slowing the hydrogen-potassium adenosine triphosphatase (H+, K+-ATPase) pump on the surface of the parietal cells of the stomach. It is used mainly for treatment of duodenal ulcer disease and H. pylori infection. It comes in a delayed-release capsule that is to be swallowed whole and taken before meals. The others are examples of H2 receptor antagonists. Page 1264

A nursing student notes that a client has a new physician order for a proton-pump inhibitor. The student knows that a frequently prescribed proton-pump inhibitor of gastric acid is a) Pepcid -famotidine b) Zantac -Ranitidine c) Nexium -Esomeprazole d) Tagamet-cimetidine

Nexium Explanation: Nexium decreases gastric acid secretion by slowing the hydrogen-potassium adenosine triphosphatase (H+, K+-ATPase) pump on the surface of the parietal cells of the stomach. It is used mainly for treatment of duodenal ulcer disease and H. pylori infection. It comes in a delayed-release capsule that is to be swallowed whole and taken before meals. The others are examples of H2 receptor antagonists. Page 1264

Peptic ulcer disease occurs more frequently in people with which blood type? a) O b) AB c) B d) A

O Explanation: pg 1266 People with blood type O are more susceptible to peptic ulcers than those with blood type A, B, or AB.

The nurse is conducting a community health education program on obesity. The nurse includes which of the following diseases/disorders in the program? a) Chronic obstructive pulmonary disease b) Oral cancer c) Rheumatoid arthritis d) Obstructive sleep apnea

Obstructive sleep apnea Correct Explanation: The nurse includes that obstructive sleep apnea is a disease/disorder associated with obesity as well as asthma; breast, endometrial, prostate, renal, colon, and gallbladder cancer; osteoarthritis, coronary artery disease, cholecystitis, cholelithiasis, chronic back pain, diabetes, hypertension, coronary artery disease, heart failure, and pulmonary embolism. Page 1272

Which of the following medications is classified as a proton pump inhibitor (PPI)? a) Omeprazole b) Ranitidine c) Cimetidine d) Famotidine

Omeprazole Correct Explanation: Omeprazole is classified as a PPI. Ranitidine, Cimetidine, and Famotidine are classified as H2 receptor antagonists. Page 1264

Which of the following represents the medication classification of a proton (gastric acid) pump inhibitor? a) Famotidine (Pepcid) b) Metronidazole (Flagyl) c) Omeprazole (Prilosec) d) Sucralfate (Carafate)

Omeprazole (Prilosec) Correct Explanation: Omeprazole decreases gastric acid by slowing the hydrogen-potassium adenosine triphosphatase pump on the surface of the parietal cells. Sucralfate is a cytoprotective drug. Famotidine is a histamine-2 receptor antagonist. Metronidazole is an antibiotic, specifically an amebicide. Page 1167

A nurse is caring for a patient hospitalized with an exacerbation of chronic gastritis. What health promotion topic should the nurse emphasize? a) Safe technique for self-suctioning b) Strategies for avoiding irritating foods and beverages c) Techniques for positioning correctly to promote gastric healing d) Strategies for maintaining an alkaline gastric environment

Strategies for avoiding irritating foods and beverages Correct Explanation: Measures to help relieve pain include instructing the patient to avoid foods and beverages that may be irritating to the gastric mucosa and instructing the patient about the correct use of medications to relieve chronic gastritis. An alkaline gastric environment is neither possible nor desirable. There is no plausible need for self-suctioning. Positioning does not have a significant effect on the presence or absence of gastric healing. - Page 1263

Which of the following medications used for the treatment of obesity prevents the absorption of triglycerides? a) Sibutramine hydrochloride (Meridia) b) Bupropion hydrochloride (Wellbutrin) c) Orlistat (Xenical) d) Fluoxetine hydrochloride (Prozac)

Orlistat (Xenical) Correct Explanation: Orlistat (Xenical) prevents the absorption of triglycerides. Side effects of Xenical may include increased bowel movements, gas with oily discharge, decreased food absorption, decreased bile flow, and decreased absorption of some vitamins. Bupropion hydrochloride (Wellbutrin) is an antidepressant medication. Sibutramine hydrochloride (Meridia) inhibits the reuptake of serotonin and norepinephrine. Meridia decreases appetite. Fluoxetine hydrochloride (Prozac) has not been approved by the FDA for use in the treatment of obesity. (less) Reference: Hinkle, J.L., and Cheever, K.H. Brunner & Suddarth's Textbook of Medical-Surgical Nursing, 13th ed. Philadelphia: Lippincott Williams & Wilkins, 2014, Chapter 47: Management of Patients With Gastric and Duodenal Disorders, p. 1272. Chapter 47: Management of Patients With Gastric and Duodenal Disorders - Page 1272

The nurse is conducting a community education class on gastritis. The nurse includes that chronic gastritis caused by Helicobacter pylori is implicated in which of the following diseases/conditions? a) Peptic ulcers b) Systemic infection c) Pernicious anemia d) Colostomy

Peptic ulcers Correct Explanation: Chronic gastritis caused by Helicobacter pylori is implicated in the development of peptic ulcers. Chronic gastritis is sometimes associated with autoimmune disease, such as pernicious anemia, but not as a cause of the anemia. Chronic gastritis is not implicated in system infections and/or colostomies. Page 5

Diagnostic imaging and physical assessment have revealed that a patient with peptic ulcer disease has suffered a perforated ulcer. The nurse recognizes that emergency interventions must be performed as soon as possible in order to prevent the development of what complication? a) Gastritis b) Peritonitis c) Acute pancreatitis d) Gastroesophageal reflux

Peritonitis Explanation: Perforation is the erosion of the ulcer through the gastric serosa into the peritoneal cavity without warning. Chemical peritonitis develops within a few hours of perforation and is followed by bacterial peritonitis. Gastritis, reflux, and pancreatitis are not acute complications of a perforated ulcer. Page 1270

The nurse is teaching a client with peptic ulcer disease who has been prescribed misoprostol (Cytotec). What information from the nurse would be most accurate about misoprostol? a) Works best when taken on an empty stomach b) Decreases mucus production c) Increases the speed of gastric emptying d) Prevents ulceration in clients taking nonsteroidal anti-inflammatory drugs (NSAIDs)

Prevents ulceration in clients taking nonsteroidal anti-inflammatory drugs (NSAIDs) Correct Explanation: Misoprostol is a synthetic prostaglandin that protects the gastric mucosa against ulceration and is used in clients who take NSAIDs. Misoprostol should be taken with food. It does not improve emptying of the stomach, and it increases (not decreases) mucus production. Page 1264

Which of the following are signs/symptoms of perforation? a) Soft abdomen b) Bradycardia c) Sudden, severe upper abdominal pain d) Hypertension

Sudden, severe upper abdominal pain Correct Explanation: Signs and symptoms related to perforation include sudden, sever upper abdominal pain, vomiting, fainting, extremely tender and rigid (board-like) abdomen, and hypotension and tachycardia, indicating shock. Page 1271

A nurse is preparing to discharge a client newly diagnosed with peptic ulcer disease. The client's diagnostic test results were positive for H. pylori bacteria. The doctor has ordered the "triple therapy" regimen. Which of the following is the correct representation of "triple therapy" refers? a) Proton-pump inhibitor and two antibiotics b) H2-receptor antagonist, proton-pump inhibitor, and an antibiotic c) Proton-pump inhibitor, an antibiotic, and Pepto-Bismol d) H2-receptor antagonist and two antibiotics

Proton-pump inhibitor and two antibiotics Explanation: Currently, the most commonly used therapy for peptic ulcers is a combination of antibiotics, proton-pump inhibitors, and bismuth salts that suppress or eradicate H. pylori bacteria. Recommended therapy for 10 to 14 days includes triple therapy with two antibiotics (eg, metronidazole [Flagyl] or amoxicillin [Amoxil] and clarithromycin [Biaxin]) plus a proton-pump inhibitor (eg, lansoprazole [Prevacid], omeprazole [Prilosec], or rabeprazole [Aciphex]), or quadruple therapy with two antibiotics (metronidazole and tetracycline) plus a proton-pump inhibitor and bismuth salts (Pepto-Bismol). Research is being conducted to develop a vaccine against H. pylori. (less)

Which of the following would be the highest priority, immediate nursing intervention for a client just diagnosed with acute gastritis? a) Recommending bed rest b) Educating the client regarding nutrition c) Advising the client not to smoke d) Providing physical and emotional support

Providing physical and emotional support Explanation: For the client with newly diagnosed acute gastritis, the priority nursing interventions are to provide physical and emotional support and help manage symptoms, which may include nausea, vomiting, heartburn, and fatigue. Teaching regarding nutrition and smoking cessation would be longer-term interventions, not immediate foci. Also, the nurse would not recommend bed rest. Page 1268

Which of the following is the most successful treatment for gastric cancer? a) Chemotherapy b) Removal of the tumor c) Palliation d) Radiation

Removal of the tumor Correct Explanation: There is no successful treatment for gastric carcinoma except removal of the tumor. If the tumor can be removed while it is still localized to the stomach, the patient may be cured. If the tumor has spread beyond the area that can be excised, cure is less likely. Page 1282

Which of the following is included as a dietary guideline for a patient who has had bariatric surgery? a) Eat four small meals a day. b) Restrict total meal size to less than 1 cup. c) Limit soda to 1 can per day. d) Make sure to drink liquids while eating.

Restrict total meal size to less than 1 cup. Explanation: Dietary guidelines would include a restriction of total meal size to less than 1 cup. The patient should eat three meals per day. Avoid liquid calories such as soda. The patient should drink plenty of water, from 90 minutes after each meal to 15 minutes before the next meal. Page 1276

Which of the following medications used for obesity improves cardiovascular disease risk factors in obese patients with metabolic syndrome? study guide a) Sibutramine (Meridia) b) Alli c) Rimonabant (Acomplia) d) Orlistat (Xenical)

Rimonabant (Acomplia) Explanation: Acomplia is the newest medication used to treat obesity. It stimulates weight reduction and improves cardiovascular disease risk factors in obese patients with metabolic syndrome. Meridia was recently pulled from the market because of the increased risk of heart attack and stroke associated with this medication. Orlistat, available by prescription and over the counter as Alli, reduces caloric intake by binding to gastric and pancreatic lipase to prevent digestion of fats. (less) Page 1273

A client is recovering from gastric surgery. Toward what goal should the nurse progress the client's enteral intake? a) Six small meals and 120 mL fluid daily b) Three meals and three snacks and 120 mL fluid daily c) Six small meals daily with 120 mL fluid between meals d) Three meals and 120 ml fluid daily

Six small meals daily with 120 mL fluid between meals Correct Explanation: After the return of bowel sounds and removal of the nasogastric tube, the nurse may give fluids, followed by food in small portions. Foods are gradually added until the client can eat six small meals a day and drink 120 mL of fluid between meals. - Page 1276

Which of the following is an accurate statement regarding gastric cancer? a) Females have a higher incidence of gastric cancers than males. b) A diet high in smoked foods and low in fruits and vegetables may decrease the risk of gastric cancer. c) Most gastric cancer deaths occur in people younger than 40 years. d) The incidence of cancer of the stomach continues to decrease in the United States.

The incidence of cancer of the stomach continues to decrease in the United States. Correct Explanation: While the incidence continues to decrease, gastric cancer still accounts for 12,800 deaths annually. While gastric cancer deaths occasionally occur in younger people, most occur in people older than 40 years of age. Males have a higher incidence of gastric cancers than females. More accurately, a diet high in smoked foods and low in fruits and vegetables may increase the risk of gastric cancer. Page 1278

Which of the following is an accurate statement regarding gastric cancer? a) A diet high in smoked foods and low in fruits and vegetables may decrease the risk of gastric cancer. b) Most gastric cancer deaths occur in people younger than 40 years. c) The incidence of cancer of the stomach continues to decrease in the United States. d) Females have a higher incidence of gastric cancers than males.

The incidence of cancer of the stomach continues to decrease in the United States. Explanation: While the incidence continues to decrease, gastric cancer still accounts for 12,800 deaths annually. While gastric cancer deaths occasionally occur in younger people, most occur in people older than 40 years of age. Males have a higher incidence of gastric cancers than females. More accurately, a diet high in smoked foods and low in fruits and vegetables may increase the risk of gastric cancer. Page 1278

A patient has recently received a diagnosis of gastric cancer; the nurse is aware of the importance of assessing the patient's level of anxiety. Which of the following actions is most likely to accomplish this? a) The nurse gauges the patient's response to hypothetical outcomes. b) The patient is encouraged to express fears openly. c) The nurse provides detailed and accurate information about the disease. d) The nurse closely observes the patient's body language.

The patient is encouraged to express fears openly. Correct Explanation: Encouraging the patient to discuss his or her fears and anxieties is usually the best way to assess a patient's anxiety. Presenting hypothetical situations is a surreptitious and possibly inaccurate way of assessing anxiety. Observing body language is part of assessment, but it is not the complete assessment. Presenting information may alleviate anxiety for some patients, but it is not an assessment. Page 1276

A nurse is providing care for a client recovering from gastric bypass surgery. During assessment, the client exhibits pallor, perspiration, palpitations, headache, and feelings of warmth, dizziness, and drowsiness. The client reports eating 90 minutes ago. The nurse suspects: a) Dehiscence of the surgical wound b) A normal reaction to surgery c) Peritonitis d) Vasomotor symptoms associated with dumping syndrome

Vasomotor symptoms associated with dumping syndrome Correct Explanation: Early manifestations of dumping syndrome occur 15 to 30 minutes after eating. Signs and symptoms include vertigo, tachycardia, syncope, sweating, pallor, palpitations, diarrhea, nausea, and the desire to lie down. Dehiscence of the surgical wound is characterized by pain and a pulling or popping feeling at the surgical site. Peritonitis presents with a rigid, boardlike abdomen, tenderness, and fever. The client's signs and symptoms aren't a normal reaction to surgery. Page 1277

The nurse is caring for a client with chronic gastritis. The nurse monitors the client knowing that this client is at risk for which vitamin deficiency? a) Vitamin B12 b) Vitamin C c) Vitamin E d) Vitamin A

Vitamin B12 Correct Explanation: Clients with chronic gastritis from vitamin deficiency usually have evidence of malabsorption of vitamin B12 caused by the production of antibodies that interfere with the binding of vitamin B12 to intrinsic factor. However, some clients with chronic gastritis have no symptoms. Vitamins A, C, and E are not affected by gastritis. Page 1265

Which of the following statements correctly identifies a difference between duodenal and gastric ulcers? a) A gastric ulcer is caused by hypersecretion of stomach acid. b) Vomiting is uncommon in patients with duodenal ulcers. c) Malignancy is associated with duodenal ulcer. d) Weight gain may occur with a gastric ulcer.

Vomiting is uncommon in patients with duodenal ulcers. Correct Explanation: Vomiting is uncommon in patients diagnosed with duodenal ulcer. Malignancy is associated with a gastric ulcer. Weight gain may occur with a duodenal ulcer. Duodenal ulcers cause hypersecretion of stomach acid. Page 1266

The nurse recognizes that the patient diagnosed with a duodenal ulcer will likely experience a) hemorrhage. b) vomiting. c) pain 2 to 3 hours after a meal. d) weight loss.

pain 2 to 3 hours after a meal. Correct Explanation: The patient with a gastric ulcer often awakens between 1 to 2 with pain, and ingestion of food brings relief. Vomiting is uncommon in the patient with duodenal ulcer. Hemorrhage is less likely in the patient with duodenal ulcer than the patient with gastric ulcer. The patient with a duodenal ulcer may experience weight gain. Page 1266

Choice Multiple question - Select all answer choices that apply. A client admitted for treatment of a gastric ulcer is being prepared for discharge. The client will follow a regimen of antacid therapy. Discharge teaching should include which instructions? Choose all that apply. two choices a) "You may take antacids with other medications." b) "Continue to take antacids even if your symptoms subside." c) "The antacids will make you sleepy, so do not operate machinery while taking them." d) "You may be prescribed H2-receptor antagonists for up to 1 year." e) "Be sure to take antacids with meals."

• "Continue to take antacids even if your symptoms subside." • "You may be prescribed H2-receptor antagonists for up to 1 year." Explanation: The client is advised to adhere to and complete the medication regimen to ensure complete healing of the ulcer. Because most clients become symptom-free within 1 week, the nurse stresses the importance of following the prescribed regimen so that the healing process can continue uninterrupted and the return of chronic ulcer symptoms can be prevented. Maintenance dosages of H2-receptor antagonists are usually recommended for 1 year. Taking antacids concomitantly with other drugs should be avoided. For best results antacids should be taken 1 hour before or 2 hours after meals. Antacids will not make the client sleepy. Page 1265

Choice Multiple question - Select all answer choices that apply. A client has a family history of stomach cancer. Which of the following factors would further increase the client's risk for developing gastric cancer? Select all that apply. a) Female gender b) Previous infection with H. pylori c) High intake of fruits and vegetables d) Caucasian ancestry e) Age 55 years

• Age 55 years • Previous infection with H. pylori The typical client with gastric cancer is between 40 and 70 years, but gastric cancer can occur in younger people. Men have a higher incidence of gastric cancer than women. Native Americans, Hispanic Americans, and African Americans are twice as likely as Caucasian Americans to develop gastric cancer. A diet high in smoked, salted, or pickled foods and low in fruits and vegetables may increase the risk of gastric cancer. Other factors related to the incidence of gastric cancer include chronic inflammation of the stomach, H. pylori infection, pernicious anemia, smoking, achlorhydria, gastric ulcers, subtotal gastrectomy more than 20 years ago, and genetics. - Page 1278

Choice Multiple question - Select all answer choices that apply. A client has recently been diagnosed with gastric cancer. He has a history of tobacco use and was diagnosed 10 years ago with pernicious anemia. He and his family are shocked about the possibility of this diagnosis because he has been asymptomatic prior to recent complaints of pain and multiple gastrointestinal symptoms. On palpation, the nurse notes two signs that confirm metastasis to the liver. Which of the following are signs? Choose all that apply. a) Hepatomegaly b) Sister Mary Joseph's nodules c) Petechiae at the palpation site d) Distented bladder e) Ascites

• Ascites • Hepatomegaly Explanation: The physical examination is usually not helpful in detecting the cancer because most early gastric tumors are not palpable. Advanced gastric cancer may be palpable as a mass. Ascites and hepatomegaly (enlarged liver) may be apparent if the cancer cells have metastasized to the liver. Palpable nodules around the umbilicus, called Sister Mary Joseph's nodules, are a sign of a GI malignancy, usually a gastric cancer. A distended bladder is not significant. Petechiae at the palpation site is a distractor for the question. (less)

Choice Multiple question - Select all answer choices that apply. A client has given a confirmed diagnosis of gastric cancer. Two more procedures may be performed to assess tumor depth and lymph node involvement and surgical resectability. Which two are the procedures? Choose the two that apply. a) Esophagogastroduodenoscopy (EGD) b) Computed tomography (CT) c) Barium x-ray of the upper GI tract d) Endoscopic ultrasound

• Computed tomography (CT) • Endoscopic ultrasound Explanation: Esophagogastroduodenoscopy for biopsy and cytologic washings is the diagnostic study of choice, and a barium x-ray examination of the upper GI tract may also be performed. Endoscopic ultrasound is an important tool to assess tumor depth and any lymph node involvement. Pelvic ultrasound is not used to confirm the diagnosis of gastric cancer. (less

Choice Multiple question - Select all answer choices that apply. A client with acute gastritis asks the nurse what might have caused the problem. Which of the following are possible causes? a) Excessive alcohol intake b) Drinking fruit juices c) Radiation therapy d) Overuse of allergy medicine e) Dietary indiscretion

• Dietary indiscretion • Excessive alcohol intake • Radiation therapy Explanation: Possible causes of gastritis include dietary indiscretion, overuse of aspirin and other nonsteroidal anti-inflammatory drugs, excessive alcohol intake, bile reflux, and radiation therapy. Allergy medicine and fruit juices are not causes of acute gastritis. (less) Page 1262

Choice Multiple question - Select all answer choices that apply. A client with active schizophrenia has developed acute gastritis after ingesting a strongly alkaline solution during a psychotic episode. Corrosion is extensive. Which of the following emergency treatments might the team working with this client use? Select all that apply. a) Syrup of ipecac b) Diluted lemon juice c) Aluminum hydroxide d) Diluted vinegar e) Gastric lavage

• Diluted lemon juice • Diluted vinegar Explanation: Emergency treatment consists of diluting and neutralizing the offending agent. To neutralize acids, common antacids (eg, aluminum hydroxide) are used; to neutralize an alkali, diluted lemon juice or diluted vinegar is used. If corrosion is extensive or severe, emetics such as syrup of ipecac and lavage are avoided because of the danger of perforation and damage to the esophagus. Page 1263

Choice Multiple question - Select all answer choices that apply. The nurse is teaching a patient on preventing dysphagia after bariatric surgery. The nurse tells the patient to avoid which of the following? Select all that apply. a) Peas b) Cheese c) Doughy bread d) Steak

• Doughy bread • Steak Explanation: The nurse should include in the teaching the avoidance of eating tough foods, such as steak as well as doughy bread. Patients should also be instructed to eat slowly and to chew their food thoroughly. Page 1277

Choice Multiple question - Select all answer choices that apply. The nurse is caring for a client who has just returned from the PACU after surgery for peptic ulcer disease. For what potential complications does the nurse know to monitor? Select all that apply. a) Perforation b) Inability to clear secretions c) Pyloric obstruction d) Hemorrhage e) Penetration f) Cachexia

• Hemorrhage • Perforation • Penetration • Pyloric obstruction Explanation: Potential complications may include hemorrhage, perforation, penetration, and pyloric obstruction. A client who has had surgery for peptic ulcer disease may have a decreased appetite in the immediate postoperative stage, but it is not something the nurse would monitor for and would not cause cachexia. Inability to clear secretions is generally not a complication of peptic ulcer surgery. (less) - Page 1268

Choice Multiple question - Select all answer choices that apply. The nurse is caring for a client who has just returned from the PACU after surgery for peptic ulcer disease. For what potential complications does the nurse know to monitor? Select all that apply. a) Penetration b) Inability to clear secretions c) Perforation d) Pyloric obstruction e) Hemorrhage f) Cachexia

• Hemorrhage • Perforation • Penetration • Pyloric obstruction Explanation: Potential complications may include hemorrhage, perforation, penetration, and pyloric obstruction. A client who has had surgery for peptic ulcer disease may have a decreased appetite in the immediate postoperative stage, but it is not something the nurse would monitor for and would not cause cachexia. Inability to clear secretions is generally not a complication of peptic ulcer surgery. - Page 1268

Choice Multiple question - Select all answer choices that apply. A nurse practitioner is considering pharmacological treatment options for a client with gastritis. Which of the following might limit the use of metronidazole (Flagyl) for this client? Select all that apply. a) Diet high in dairy products b) Current use of warfarin (Coumadin) c) Occupation as a landscape architect d) History of alcoholism e) Allergy to penicillin

• History of alcoholism • Current use of warfarin (Coumadin) Explanation: Clients who take metronidazole (Flagyl) should avoid alcohol. Flagyl also increases the blood-thinning effects of warfarin (Coumadin). Amoxicillin (Amoxil) is contraindicated in clients allergic to penicillin. A diet high in dairy products would limit the use of metronidazole (Flagyl). Tetracylcine might be problematic for those who work outdoors, because it can impose a photosensitivity reaction--clients taking it would need to use sunscreen. Page 1264

Choice Multiple question - Select all answer choices that apply. Which of the following dietary guidelines should be followed following bariatric surgery? Select all that apply. a) Eat slowly. b) Include two protein snacks per day. c) Total meal size should be 1 cup. d) Eat and drink at the same time. e) Eat three meals per day.

• Include two protein snacks per day. • Eat slowly. • Eat three meals per day. Explanation: Dietary guidelines for the patient who has had bariatric surgery include eating slowly, eating three meals per day, and including two protein snacks per day. The patient should avoid eating and drinking at the same time and his or her total meal size should be less than 1 cup. Page 1276

Choice Multiple question - Select all answer choices that apply. A nursing student is caring for a client with gastritis. Which of the following would the student recognize as a common cause of gastritis? Choose all that apply. a) DASH diet b) Irritating foods c) Overuse of aspirin d) Participation in highly competitive sports e) Ingestion of strong acids

• Ingestion of strong acids • Irritating foods • Overuse of aspirin Explanation: Acute gastritis is often caused by dietary indiscretion—a person eats food that is irritating, too highly seasoned, or contaminated with disease-causing microorganisms. Other causes of acute gastritis include overuse of aspirin and other nonsteroidal anti-inflammatory drugs (NSAIDs), excessive alcohol intake, bile reflux, and radiation therapy. A more severe form of acute gastritis is caused by the ingestion of strong acid or alkali, which may cause the mucosa to become gangrenous or to perforate. A DASH diet is an accronym for Dietary Approaches to Stop Hypertension, which would not cause gastritis. Participation in competitive sports also would not cause gastritis. Page 1262

Choice Multiple question - Select all answer choices that apply. A 66-year-old African-American client has recently visited a physician to confirm a diagnosis of gastric cancer. The client has a history of tobacco use and was diagnosed 10 years ago with pernicious anemia. He and his family are shocked about the possibility of cancer because he was asymptomatic prior to recent complaints of pain and multiple gastrointestinal symptoms. On the basis of knowledge of disease progression, the nurse assumes that organs adjacent to the stomach are also affected. Which of the following organs may be affected? Choose all that apply. a) Liver b) Duodenum c) Lungs d) Pancreas e) Bladder

• Liver • Pancreas • Duodenum Explanation: Most gastric cancers are adenocarcinomas; they can occur anywhere in the stomach. The tumor infiltrates the surrounding mucosa, penetrating the wall of the stomach and adjacent organs and structures. The liver, pancreas, esophagus, and duodenum are often already affected at the time of diagnosis. Metastasis through lymph to the peritoneal cavity occurs later in the disease. (less)

Choice Multiple question - Select all answer choices that apply. A client is prescribed a histamine (H2)-receptor antagonist. The nurse understands that the following are H2-receptor antagonists. Choose all that apply. a) Lansoprazole (Prevacid) b) Cimetidine (Tagamet) c) Famotidine (Pepcid) d) Esomeprazole (Nexium) e) Nizatidithene (Axid) f) Ranitidine (Zantac)

• Nizatidithene (Axid) • Ranitidine (Zantac) • Famotidine (Pepcid) • Cimetidine (Tagamet) Explanation: H2-receptor antagonists suppress secretion of gastric acid, alleviate symptoms of heartburn, and assist in preventing complications of peptic ulcer disease. These medications also suppress gastric acid secretions and are used in active ulcer disease, erosive esophagitis, and pathological hypersecretory conditions. The other medications listed are proton-pump inhibitors. (less) Page 1264

Choice Multiple question - Select all answer choices that apply. A nurse is assessing a client who is interested in undergoing bariatric surgery. Which of the following are criteria for candidacy? Select all that apply. two choices a) No history of psychiatric illness b) Body mass index (BMI) of 27 c) Ability to explain why other weight-loss attempts have failed d) Understanding of the causes of obesity e) Understanding of required lifestyle changes

• No history of psychiatric illness • Understanding of required lifestyle changes Explanation: A client who is interested in bariatric surgery must be evaluated by a mental health practitioner to ensure that he or she does not have serious psychiatric disorders and is motivated to comply with lifestyle changes related to eating patterns, dietary choices, and elimination. The client also must be morbidly obese. A client with a BMI of 27 is overweight, but not morbidly obese. While assessment of knowledge about causes of obesity and its associated risks as well as insight into the reasons why previous diets have been ineffective are included in the client's plan of care, these do not predict positive client outcomes following bariatric surgery. Page 1273

Choice Multiple question - Select all answer choices that apply. Which of the following manifestations are associated with a deficiency of vitamin B12? Select all that apply. a) Macrocytic anemia b) Pernicious anemia c) Loss of hair d) Thrombocytopenia e) Lethargy

• Pernicious anemia • Macrocytic anemia • Thrombocytopenia Explanation: Decreased vitamin B12 can result in pernicious anemia, macrocytic anemia, and thrombocytopenia. Decreased iron can result in lethargy and loss of hair. Page 1280

Choice Multiple question - Select all answer choices that apply. A client with peptic ulcer disease wants to know nonpharmacological ways that he can prevent recurrence. Which of the following measures would the nurse recommend? Select all that apply. a) Avoidance of alcohol b) Substitution of coffee with decaffeinated products c) Following a regular schedule for rest, relaxation, and meals d) Smoking cessation e) Eating whenever hungry

• Smoking cessation • Following a regular schedule for rest, relaxation, and meals • Avoidance of alcohol Correct Explanation: The likelihood of recurrence is reduced if the client avoids smoking, coffee (including decaffeinated coffee) and other caffeinated beverages, and alcohol. It is important to counsel the client to eat meals at regular times and in a relaxed setting and to avoid overeating. Page 1268

Choice Multiple question - Select all answer choices that apply. Which of the following interventions are appropriate for clients with gastritis? Select all that apply. a) Use a calm approach to reduce anxiety. b) Give the client food and fluids every 4 hours. c) Provide general education about how to prevent recurrences. d) Discourage cigarette smoking. e) Notify the physician of inidicators of hemorrhagic gastritis.

• Use a calm approach to reduce anxiety. • Discourage cigarette smoking. • Notify the physician of inidicators of hemorrhagic gastritis. Explanation: The nurse should use a calm approach when answering questions and providing teaching. He or she should discuss smoking cessation and monitor for any indicators of hemorrhagic gastritis. The client will take nothing by mouth for up to a few days until symptoms subside. The nurse needs to develop an individualized teaching plan for the client that includes information about stress management, diet, and medications. (less) Page 1263


Set pelajaran terkait

Review of Learning Objectives with Selected Key Terms - Chapter 1 - Addressing Environmental Problems

View Set

ch 3 preliminary considerations and procedures

View Set

APUSH Vocabulary Chapter 18 & 19

View Set